formulas of creativity

50
1 Геннадий Иванов ФОРМУЛЫ ТВОРЧЕСТВА, ИЛИ КАК НАУЧИТЬСЯ ИЗОБРЕТАТЬ Gennady Ivanov FORMULAS OF CREATIVITY OR HOW TO BECOME AN INVENTOR * Translated from Russian by Marija Dobrovolska-Stoian Alexander Bykovsky Jelena Volkova * In various Russian sources found translated as “THE FORMULAS OF CREATIVITY OR HOW TO LEARN TO INVENT

Upload: surya-aji

Post on 14-Jan-2016

37 views

Category:

Documents


4 download

DESCRIPTION

Formulas of Creativity or How to be an Inventor - Gennady Ivanov

TRANSCRIPT

Page 1: Formulas of Creativity

1

Геннадий Иванов

ФОРМУЛЫ ТВОРЧЕСТВА,

ИЛИ КАК НАУЧИТЬСЯ ИЗОБРЕТАТЬ

Gennady Ivanov

FORMULAS OF CREATIVITY OR HOW TO BECOME AN INVENTOR*

Translated from Russian by

Marija Dobrovolska-Stoian Alexander Bykovsky Jelena Volkova

* In various Russian sources found translated as “THE FORMULAS OF CREATIVITY OR HOW TO LEARN TO INVENT”

Page 2: Formulas of Creativity

2

Questions and suggestions: [email protected] The original russian edition written by Gennady Ivanov was published in 1994 in Russia at the publishing house „Просвещение“ („Prosveschenije“) with the title »Формулы творчества, или как научиться изобретать« Copyright © 2013 by Dr. Robert Adunka Conditions of sale All rights reserved. No part of this publication may be reproduced or transmitted in any form or by any means, electronic or mechanical, including photocopy, recording, or any information storage and retrieval system, without permission in writing from the publisher. All used pictures are made by the publisher. Used photocopies are not protected under copyright law or other legislation governing the protection of intellectual property. Translation: Marija Dobrovolska-Stoian, Alexander Bykovsky, Jelena Volkova Editing: Dr. Robert Adunka, Alexander Munk German publisher: Dr. Robert Adunka Illustrations: Alexander Munk Setting: Alexander Munk

Page 3: Formulas of Creativity

3

CONTENTS

CONTENTS .................................................................................................................................. 3 AUTHOR’S INTRODUCTION ....................................................................................................... 5 WHAT DOES HUMANKIND NEED CREATIVITY FOR? ............................................................. 7 TECHNOLOGY OF CREATIVITY .............................................................................................. 10 LOOKING FOR THE METHOD .................................................................................................. 14 SYSTEM ANALYSIS .................................................................................................................. 22 ... AND START TO INVENT ....................................................................................................... 28 IN THE BEGINNING WAS A CONTRADICTION ....................................................................... 41 COMPETITION WITH CONTRADICTIONS ............................................................................... 56 INVENTIONS BY FORMULAS ................................................................................................... 67 GO AHEAD, STANDARTS ........................................................................................................ 79 EVERYTHING I HAVE – I’M CARRYING WITH ME .................................................................. 83 AT THE PSYCHOLOGICAL EDGE ........................................................................................... 87 LAWS OF DEVELOPMENT OF THE TECHNICAL SYSTEMS ................................................. 97 IN CHASE OF THE LIGHT ....................................................................................................... 119 MAIN STAGES OF THE DEVELOPMENT OF THE TECHNICAL SYSTEM (Fragments) ..... 125 MYSTERIES OF AGES ............................................................................................................ 127 IN HARMONY WITH NATURE ................................................................................................. 140 INSTEAD OF CONCLUSION ................................................................................................... 150 HINTS ....................................................................................................................................... 152 SOLUTIONS ............................................................................................................................. 156 APPENDICES ........................................................................................................................... 163 RECOMMENDED LITERATURE ............................................................................................. 195

Page 4: Formulas of Creativity

4

Page 5: Formulas of Creativity

5

Gennady Ivanov

AUTHOR’S INTRODUCTION

Do you know how powerful a man can be?

Fyodor Dostoyevsky

This book is meant for those who want to become an inventor, who is passionate about the world of technical creativity and who strives to get to know it deeper. The aim of this book is to help you develop your inventing skills.

The path to invention is hard, it is impossible to conquer it without belief in one’s own power, without knowledge that humanity has been collecting throughout many centuries of its history. At the dawn of human development inventors were seen as gods or mythical heroes. Remember Prometheus, Daedalus, Icarus and other heroes from the Ancient Greek

mythology, who invented fire and wings, tools for fishing, potter’s wheel, carpenter’s tools and many other objects useful for human beings.

The powerful Heracles multiplied his fame by, putting it in modern language, using hydrocleaning of dung in closed animal farming facilities. He led the roaring waters of the nearby river into the terribly neglected Augeia’s horse stables and cleaned them out perfectly within a few hours.

Still, the troop of these “divine” investors was small; it obviously didn’t cope with all the growing human needs. So the humans had to deal with divine things themselves. So humanity entered the era of scientific and technological revolution by pushing a fire-breathing monster in front of it. The world started changing rapidly getting filled by the smell of engine oil, machine and jet airplane roaring.

People from the Middle Ages would be very surprised if they had a chance to walk down the streets of a modern city. There would be exclamations of surprise just looking at an electric bulb or talking on the phone to someone who is many hundreds or even thousands of kilometers away. TV, radio, cinema, car, plane – all this would be really hard to grasp for them. Yes, we can do many things today. But our offsprings will hardly be able to explain many of our actions. For instance, why do we destroy the most precious fossils – coal or crude oil, burning them in ovens in our homes or in various engines, while having an ocean of sun or cosmic energy, and beneath us – the boiling magma of the Earth. The biggest surprise will be probably caused by our mismanagement of using the main energy of the Earth: the mental one. In our scientific and technical actions to them we will look like children who are fumbling with a kaleidoscope, hoping to find a new beautiful design.

Yes, we have changed many things but we haven’t dealt with the technology of creativity. We are still looking for new things using the trial and error method, as we did hundred, thousand and tens of thousands years ago. We are using approximately the same method as our ancestor was looking for an appropriate stone for his club. He took it, tried it on, threw it away and started looking for another one. Looking for something new we keep doing the same steps: we take something, try it and throw it away. We take something again, try it…

Such an approach to the search of the new can’t be acceptable during the era of scientific and technological progress as it’s connected with low effectiveness and a huge loss of time. Genrich

Page 6: Formulas of Creativity

6

Saulovitch Altshuller (a.k.a. sci-fi writer G. Altov) was the first person in the world to discover that the technology which is being created by humans for their needs, is not developing randomly or according to the wishes of the most talented inventors but has certain rules and objective laws which one has to know and apply. This thought was first formulated by Altshuller more than 30 years ago. Later on the author developed these first statements into a number of books and scientific papers. So the science about creativity was born, which was later called the Theory of Inventive Problem Solving – TRIZ.

Today a big pool of researchers are still developing TRIZ, finding new objective laws not just in technical but also in biological, social, art and other systems. Based on all this a General Theory of Powerful Thinking can be created.

Thanks to TRIZ it became possible to source many creative elements out to machines. In the Minsk laboratory guided by V. M. Tsurikov an Inventing Machine was created which multiplies the human possibilities by hundreds of times. And this is just the beginning, just the first steps towards a creative revolution which is brought by TRIZ.

The production of material goods requires the production of new ideas in the first place. This is why TRIZ is being taught at many hundreds of schools in Russia. As a result creativity stops being the skill of the few. Having mastered the basics of TRIZ, practically anybody can become an inventor.

This book is dedicated to this topic. The examples mentioned here are real tasks, many of them were solved by the author, who in the past has never thought about inventing, and by his friends and colleagues, neither of whom were born as Edisons’.

Besides, tasks were included into this book, which were solved by pupils aged 13-14, many of whom have later become real inventors. There are regular children who have mastered the methods of creative thinking and so entered a new and wonderful world. They felt that they can change the world for the better and participate in empowering the scientific and technological progress. Mainly this book contains tasks which have been solved by the members of the Young Inventors’ Workshop “Impulse” which I had guided for many years in Angarsk. Workshop members have won numerous prizes at inventing contests; have made real, “adult”, officially registered inventions.

Unfortunately the volume of this book did not allow the author to include many interesting sections of TRIZ in it. Many laws of development of technical systems and TRIZ theses have remained unexplained. A little comforting may sound the words of the ancient philosopher Socrates regarding his first getting to know a new science, “What I managed to learn is great. I think, so is what I still have to learn”.

Answers to many questions related to theory and practice of the inventing art, you can find in the literature that is listed in the end of this book.

The author will consider his task fulfilled, if you will become interested in inventing and will wish to participate in the technological progress.

Finally, I would like to express my sincere acknowledgement to the wonderful person who dedicated his life to exploring the laws of creative thinking, who opened new horizons of possibilities to the humanity, the author of TRIZ – Genrich Saulovitch Altshuller.

I would also like to say a sincere thank you to my friends and colleagues: M K. Bdulenko, A. A. Bystritsky, I. M. Vertkin, V. I. Volosyany, I. B. Buhman, M. I. Deniskin, S. I. Ivanovksky, B. L. Zlotin, A. N. Limarenko, S. S. Litvin, A. B. Selyutsky, A. Sayfutdinov, Y. P. Salamatov, K. A. Sklobovsky, A. I. Tyutrin, M. I. Sharapov and many others whose help has determined the appearance of this book significantly.

Page 7: Formulas of Creativity

7

WHAT DOES HUMANKIND NEED CREATIVITY FOR?

The human has been obviously created to think: this is his quality, his whole credit;

his duty is in right thinking...

Blaise Pascal

For more than two hundred millions of years the Sun has been moving around the center of the Galaxy. This is the so-called galaxy year. Just a “year” ago was the Mesozoic era, the Age of Reptiles. The history of humankind consists of less than two “days”. A regular human life lasts only a few “seconds”, using these measures. What can one manage during these seconds, even galactic ones?

Let’s look back just a few decades ago. We will be surprised to find out that our ancestors didn’t have planes or helicopters, satellites or hydropower stations. There was no radio, TV, phone, CD-player or nylon and many thousands of things that surround us today and which we cannot imagine out of our life. Moreover, according to the theory of unity of the living and lifeless in nature, formulated by academician V. I. Vernadsky, the biosphere around us is changing its quality to get a new state, noosphere, under the influence of human social and scientific activity. V. I. Vernadsky underlines, “The human being has understood for the first time, really, that he is an inhabitant of the planet and can – and should think and act in the new aspect, not just in the aspect of a singular person, family, community, state or their unity, but also in the planetary aspect”.

Cognizing the laws of nature and using them sensibly, the human being is becoming the biggest nature forming power which can be compared to geological processes. He brings new kinds of substance and energy exchange into nature, exploring the space, and is involved into things happening in it.

“Maybe there is some kind of goal on Earth, which humanity is striving for, which consists only in endless movement towards the goal, so to say, in life itself…“wrote F. Dostoyevsky more than 100 years ago. Yes, our life is a never ending search, indeed.

The human being seems to be permanently busy with changing something, improving something. Why does he need it for? Which power forces him to get rid of the old, used and tested? Why can’t we rest after we’ve fixed something ones and have a calm, peaceful life without hectics, just living the time we’ve got? Unfortunately it doesn’t work. Nature wouldn’t let us have such a way of life. Why?

Psychologists have made an interesting experiment. Lab rats were put into a big compound. There they had plenty of everything and didn’t have to search for anything. After some time the rats got nervous. The most active ones started looking at the narrow, dark corridor next to the compound. This compound led into an empty room with metal floor which had some live wires. The rats squeezed themselves through the narrow corridor, their fur stood on end, they panicked and ran out the room scared of the noise, but then they came back. What brought them to this room? Search activity!

It’s interesting to know that those rats who survived the longest in this “paradise” colony, were the most curious, who went several times to the scary, dark corridor. On the opposite, those rats, who just enjoyed their comfortable life, didn’t leave the compound, soon lost every activity, got fat and died.

Page 8: Formulas of Creativity

8

The human search activity is much more versatile and rich in content that that of animals, as for the human it has a higher value. From the biologic point of view it’s not important where the human will show his search activity – in music, science, literature, technics, social field, household etc. It’s only important that this activity is present and would be developed up to a creative level.

Creativity is not just luxury for the few, it’s a biological necessity, often remaining unrealized.

It’s a fact that people who have no interest in their work or studies, who are not passionate about anything, who avoid solving problems including creative elements, are sick more often, get old faster and die sooner.

Even in extreme conditions humans are supposed to do the same kind of things: look for an aim, set a task and solve it. Remember Jack London’s short story, “Love of Life”. A hungry, physically exhausted and almost frozen human keeps setting some concrete task to himself – reach the next hill, catch a quail, conquer some more meters. He solves plenty of tasks, difficult for his situation, which need a huge amount of imagination. And he wins life.

It’s been known that a human needs for usual life activity positive emotions to outnumber the negative ones. Creativity in any field of action, but especially in invention, is one of the most powerful and unquenchable source of positive emotions.

What happens if a person can’t, or, which hardly ever happens, doesn’t want to find elements of creativity in his life, in his work? Such people reach the thought of the dominance of evil in life and the futility of all human effort. This is the way of nature to punish those who don’t use their mind for the purpose intended.

How does creative work influence the development of the society as a whole? Let’s assume that year to year or even day to day we will lessen the volume of our creative ambitions. The formula is simple: “I’m a tiny human being; I don’t need much – the less, the better”. Graphically we could visualize is as a number of circles, the radii of which would symbolize the shrinking creative activity but the surface of the circles – the shrinking contact area with the environment, where life demands creative work.

So, if with time the radius of creative work is shrinking, so does the length of the circle, and, along with it, the amount of tasks which have to be solved. The activity of the human is getting simpler, he degrades and moves on to the way of life of primitive animals. Finally, the human turns into a unicellular creature, for instance, an amoeba, which, following the same principle, is getting more and more primitive until it turns into a nonliving molecule. There is nothing more to solve, no more questions! But there is also no life...

Let’s consider a second variant, where the radius of creative work is neither growing nor shrinking. Everything stays as it is. We don’t change anything. We live as we used to live ten, hundred, thousand and tens of thousands of years ago. The formula is as follows: “This is the way our ancestors and their ancestors used to live, that’s why I’m going to live like this as well”. Generation to generation the tasks are the same, the solutions are the same as well. The

Page 9: Formulas of Creativity

9

irritants repeat themselves and produce reflectory actions, after that innate instincts appear. Creativity disappears completely. Is it good or bad?

It’s good, because there is no new energetic input, it’s enough to listen to one’s instincts, the “inner voice” and everything will be okay. This is the way of life of most animals. The birds build extremely complicated nests, the spiders produce nets, the beauty and strength of which fascinate the scientific world. In fact, so do we do a part of our actions unconsciously, by inertia. It’s also bad, because the nature around us is not stable. With time the climate on Earth is changing, the rivers, lakes, seas disappear, others appear.

The atmosphere, the flora, fauna are changing. The whole world is changing. What about us? Even a microbe changes its population if the environment has changed. The disappearing of thousands of animal species who have been recorded in the Red List can be explained by the fact only that they haven’t been able to adjust the changes.

Can the human being afford such a tiny security of his existence? We can’t, of course. As Lewis Carroll remarked in his “Alice in Wonderland”, “it takes all the running you can do, to keep in the same place”. This constantly growing “run”, known today as the scientific technical revolution, appears to be the only method of further existence for society and raising its resilience.

So, out of the three ways of existence offered for humankind, only one is valid: growing search activity, multiplying knowledge and creative skills. If we want to live, hence, we have to learn doing it, faster, than the changes in nature take place. Graphically all the offered variants are presented in illustrations.

The later in time, the larger radius we should have to be able to answer more and more questions. The creative way of life is not a privilege of individuals, it’s the only way of normal existence and development for our society.

Page 10: Formulas of Creativity

10

TECHNOLOGY OF CREATIVITY

I’d rather not even think of finding any kind of truth than doing it

without any method.

René Descartes

WHY DO WE INVENT BADLY

Most probably you’ve been noticing that any work you had to do in most cases was done according to an elaborate plan, according to a certain technology – the rules of action. Breaking these rules leads to failure or causes additional trouble.

This is the way everybody works. A stove-setter will make a plan in his mind, before building the oven: he will make a plan of actions, from preparing the bricks and mortar and up to the very last stroke of paint when finishing the surface.

A surgeon will go over the whole operation in his mind, before starting it – from the very first cut to the final stitching.

The turner, making a piece, works according to a tool layout where the speed of the horizontal and vertical saddle feed are mentioned, rotation frequency of the spindle, the type of the knife and many more details which have to be known in order to produce a qualitative piece.

Even if you are planning to go to the cinema, you do it according to a certain “technology”. You plan the departure time from home, you think through the way to the cinema, you pick a way of transportation – on foot, by bus, tram, and, at last, you count the money – will it be enough for the ticket? If just one of these actions doesn’t match the final goal, the trip to the cinema has to be postponed.

So, technology is a ladder to the goal set. Without it no work in the world is getting done! Good or bad, some kind of technology has to be there, otherwise any activity, any work is getting useless.

Does an inventor have a technology? Until recently it was difficult to answer this question. The classical literature bears the image of an inventor, pale, exhausted by endless searches and back-breaking work. Every time he had to start from the unknown, he made hundreds and thousands of trials but was never sure that he’s doing the right thing and that the work done will lead him to success. Why? Mainly because he didn’t have any kind of technology. What has to be done first, what comes next – it was unknown. Everything was decided by chance and persistence of the inventor who kept searching for this lucky chance. The inventor, hardworking as a bee, kept browsing through the possible variants, hoping, that at some point he will succeed in something.

Of course, along with acquiring knowledge, the human being was able to connect different elements with each other in a meaningful way, but the technology of creating inventions was still unknown.

Page 11: Formulas of Creativity

11

STAKE ON CHANCE

The British chemist Ch. Macintosh happened to spill a bottle of a fluid in his lab – he spilled some fluid — solvent-naphtha on a piece of dried natural rubber. He noticed that the hard rubber became soft as fresh dough. So he had an assumption: if it would be possible to impregnate cloth with it, it would become waterproof. So waterproof raincoats were invented, along with waterproof shoes and many other things which became wildly popular very soon.

Unfortunately, natural rubber cracked in frost and melted from heat. It was necessary to remove these defects. Hundreds of experiments passed. And again, a lucky change helped, but a different inventor this time.

In 1839 C. Goodyear was making another experiment and dropped a piece of rubber and a piece of sulphur on a hot stove. The rubber and the sulphur melted together into a new elastic substance which didn’t crack in frost or melt in high temperature. So the method of rubber vulcanization was invented which has been applied producing commodities and household goods.

The engineer Samuel Brown was lying under a tree, thinking over the construction of a new bridge, which would be lighter than any of the existing ones but more solid. Suddenly… he saw a spider net between the tree branches. That was exactly what he was looking for! So the idea of a hanging bridge appeared.

The Dutchman Z. Janssen, an opticist, had polished one of his lenses and decided to look at its surface using another magnifying lense. By accident he looked through the two lenses onto a church tower far away. He was overwhelmed. It seemed that the church tower was right before his eyes, he could see every tiny detail. So the idea of a telescope was born.

The French physicist Antoine Becquerel discovered radioactivity after having found a light-struck photoplate which happened to be next to uranic salt.

Jannsen BecquerelMacintosh Jannsen BecquerelMacintosh

In 1870 A. Marillet invented the chemical cleaning of fabrics. It happened after he pulled a dirty suit out of a barrel with terpentine where it had fallen into by accident.

Owen Richardson spilled by accident some hydric dioxide on a goose feather which turned colourless. So the method was invented how a brunette could become a blonde quickly.

Page 12: Formulas of Creativity

12

The Frenchman Bernard Courtois invented iod accidently in 1811.

The alchemist Barned discovered phosphorus accidently in 1674, while actually looking for the “Philosopher’s Stone” and experimenting with human hair.

Also accidentally were invented penicillin, steel concrete, X-rays, the method of spark machining and many more.

The world is full of lucky chances! But to make sure that the chance happens, it’s necessary that it happens to a person who’s prepared, in the exact right moment when he’s solving a task, fitting the case.

Too many unclear circumstances to talk about a concrete technology of search. Only one thought appears: in order to maximize the possibility of meeting the right chance, it’s necessary to maximize the amount of one’s actions, experiments and trials.

THE RIGHT TO MAKE MISTAKES

Probably it’s the oldest method of creating technical systems and devices. This method was already used by our far ancestor who tried to tie a fitting stone to his stick.

This method is fairly simple and secure – you take it, try it, throw it away. Again and again. This is the way people used to work hundred years ago and hundred thousands years ago. The trial and error method is today probably the most widespread among inventors and innovators. Every day about 500 technical solutions are being patented and just a small part of them, not more than 10-15 per cent, are implemented in practice. The rest is something like “information noise”, i.e. these solutions will never be implemented, these are the “splashes” of chance, “garbage” left from human intellectual activity according to the trial and error method. It’s been found out that in order to make even one middle-level-invention using this method, it’d take many hundreds or even thousands of trials. This is probably the reason why the saying appeared: ”Talent is 99 per cent of sweat and 1 per cent of luck”.

Patent

---- ----------------------

500

50 - 75

patents products

Patent

---- ----------------------

Patent

---- ----------------------

500

50 - 75

patents products

That’s right, a talented inventor manages to go through tens or even hundreds of variants, in his mind, really quickly. This is hard and exhausting work, not many are able to do it physically. So, what shall we do?

Page 13: Formulas of Creativity

13

Edison

The luminary inventor A. Edison had a whole institute of experimentalists, who, for example, made many thousands of trials looking for the right material for the glowing filament for the electric bulb, testing all the available materials. They used the known metals and composites, carbonized fibres from silk, wool, Bristol board, paper and even human hair. Edison sent his employees to Brazil, China, Japan and other countries, to search and collect different kinds of plants, for example, bamboo. As trials showed, carbonized sticks made from some kinds of bamboo, worked quite well as glowing filament.

Edison got several dozens of patents to different kinds of glowing filaments for an electric bulb. But the working efficiency of these lamps was still very low. Only much later Edison discovered the reason for it: oxygen that was left in the bulb after removing the air from it, was oxidating the material of the fibre and destroyed it. High-vacuum lamps were produced or they were filled by inert gas. The working efficiency of the lamps went up. Now it was possible to make the glowing filament from

usual heat-resistant metals, which by that time have not been that rare anymore.

Edison and his employees had to make about 40.000 trials in order to get a more or less functioning alkaline accumulator. That really were Herculean efforts.

Let’s give proper respect to Edison’s talent and ambitiousness. However? if the trial and error method was tolerated in the beginning of the 20th Century, when the world of technics was just being born, then now it has used its possibilities completely. It has been calculated that in order to keep the speed of inventing activity, it would soon be necessary to put the whole population of our planet to inventor’s desks.

Yes, trials are necessary, mistakes are unavoidable and we will always have the right to make mistakes, but their amount in the world should get less, as humanity is getting more and more “adult”. Otherwise we’d have to leave the thought about the accelerating development of human civilization by means of effective use of accumulated knowledge in inventions.

Page 14: Formulas of Creativity

14

LOOKING FOR THE METHOD

A path of errors leads to great truths.

Paul Valery

The first one who had the thought about the necessity to improve the level of organization of creative work, was probably the Ancient Greek hermit and philosopher Diogenes. Having no laboratory, he chose a barrel as his refuge and staying there in complete isolation, he kept thinking about the perfection of nature and the endlessness of the universe.

Unfortunately, we don’t know whether Diogenes had any followers and whether living in a barrel improves creative skills.

The Roman poet and philosopher Titus Lucretius Carus describes in his philosophical poem “On the Nature of Things” the teachings of the Greek philosopher Epicurus who suggests creating new objects by means of combining their parts. That was a significant step forward already, that will create many sciences in the future, including combinatorics.

The famous Spanish scientist from the early Middle Ages, Raymundus Lullus, has created his own method of acquiring the truth and described it in his work named “The Ultimate General Art”. The main idea was that different notions were replaced by symbols which could be combined and interpreted. For that Lullus had created a special machine that was made from a series of discs, rotating on an axe with symbolic signs. After the discs were stopped, the matching symbols were counted and the machine gave a pretty concrete answer to any problem. Although the “mechanization” of mental work was complete, the method had not achieved remarkable distribution. Lullus himself left to a desert and never stopped perfectioning his device, for the rest of his life.

The first really applicable methods of activization of the human art process started appearing in the late 1920ies. Brainstorming, the method of focal objects, morphological analysis, the method of control questions, synectics, matrix method, integral method, associative method etc. appeared.

There are more than 30 of their kind. Let’s get to know several of the methods better.

THE METHOD OF BRAINSTORMING

This is the oldest method. Vikings have used it already, solving their problems. During their sea expeditions the whole crew came together to look for advice, everyone could bring in his ideas. First the boys and the apprentice seamen would talk and then the senior seamen and the others. The captain had the last word and he was the one who made the decision.

The modern modification of brainstorming, the so called “brain attack” was offered by the American marine officer A. Osborn, who was captain of a small cargo ship during the WW2.

Once, fearing the attack of German submarines, A. Osborn let the whole crew come together on deck and asked everyone to express his opinion on how to protect an unarmed ship from tin fish. Optimists are everywhere. A member of the crew suggested everyone to stand near the board and, when the tin fish will be approaching the ship, everyone should blow against it. The tin fish was then supposed to miss its aim. This time there was no chance to test this method of

Page 15: Formulas of Creativity

15

protection in practice. The crew had a good laugh and left to their quarters. But A. Osborn found the idea quite successful. He planted a ventilator on deck who created a powerful stream of water and during one of their voyages he really succeeded by “blowing” a tin fish away from his ship.

After the war A. Osborn started working on the detailed variant of the method. His book, „Applied Imagination“, became the basis of many lectures in various universities, scientific institutes and companies in the USA.

The point of brainstorming is to let go thoughts from the subconsciousness. According to Freud’s theory, guided consciousness is just a thin layer on the mass of unguided subconscience. During brainstorming it is necessary to create conditions to “unpack” the subconscience.

THE METHOD OF FOCAL OBJECTS

The point of the method is in placing the features of randomly chosen objects onto the object which has to be improved.

Let’s assume, you have to improve a toothbrush, using this method. You have to choose at least 3-4 random objects (from a dictionary, catalogue, book, magazine…). I picked the words “bell”, “candy” and “lamp”. Let’s make a list of these objects’ features.

Bell — electric, school, clock, bicycle...

Candy — caramel, chocolate, sweet, aromatic, vitamin...

Lamp — desk, hanging, antibacterial...

Now let’s try to connect the features we have to the object to be changed, in our mind at least – the toothbrush. What do we get? Using your imagination, you could think of a toothbrush that calls you in the morning and asks you to use it, produces a pleasant flavour. You can also bite a bit of the handle as it’s edible and has a lot of vitamins and is also antibacterial, i.e. it kills all the harmful bacteria in your mouth.

Is it a good brush?

The method of focal objects doesn’t guarantee that the result will be anything useful, but still it makes the mind go loose and sometimes it leads to unexpected combinations. This method

Page 16: Formulas of Creativity

16

helps your imagination improve but we cannot talk about any kind of direct or planned changing of the object.

A variation of this method is the method of garland from chances and associations.

THE METHOD OF SYNECTICS

The most effective of the methods of psychological creativity activation created abroad is synectics (offered by W. G. Gordon), which is the improved version of brainstorming.

During a synectical brainstorm criticism is allowed that helps develop and change the ideas expressed. This brainstorm is lead by a constant group. Its members get used to working together, don’t fear criticism anymore and don’t get offended if anyone refuses their suggestions.

In this method four kinds of analogies are introduced – direct, symbolic, fantastic and personal.

Using direct analogy, the object observed is being compared to a more or less similar object in nature or technics. For example, to improve the process of painting furniture the application of direct analogy consists in looking at how minerals, flowers, birds are coloured etc., how paper is being coloured, wallpaper, film footage, TV images etc.

Using symbolic analogy we have to formulate the phrase in a paradoxical way, summarizing the point of the phenomenon. For example, solving a task about marble, the expression “rainbow permanency” was found, as the polished marble (except the white one) is full of bright patterns which remind of the rainbow, but the patterns are consistent.

Using fantastic analogy, we have to imagine fantastic tools or characters, which or who will be doing what is needed in the task. For example, a road must be where a car’s wheels touch it.

Using personal analogy (empathy) we can imagine that we are the object which we are talking about in the task. For instance, in the example with painting furniture, we could imagine ourselves being a white crow which wants to get painted. Or, if the gear transmission is being improved, then imagine a gear wheel, which is rotating around its axis, placing the sides for the adjacent gear wheel. It is necessary, in the truest sense of the word, to enter “into the image” of this gear wheel to feel by yourself all its inconveniences or overloads. What gives such transformation? It significantly reduces the inertness of thinking and allows considering the problem from a new point of view.

Page 17: Formulas of Creativity

17

MORPHOLOGICAL ANALYSIS

Morphological analysis is an example of a system approach in the field of invention. The method was developed by the well-known Swiss astronomer Fritz Zwicky. Through this method he succeeded to find a lot of original technical solutions in the rocket production in a relatively short time.

For carrying out the morphological analysis the precise formulation of the task is necessary, and regardless of the fact that in the original problem is describing only one specific system, all possible systems with the similar structure are summarized and the answer to the more general question is given in the end. For example, it is necessary to study the morphological character of all types of vehicles and to offer a new efficient design of the device for transportation over the snow – a snowmobile.

The exact definition of the category of the involved systems (devices) can reveal the basic characteristics or parameters that facilitate the search for new solutions. The morphological features of the vehicle (snowmobile) may be functional parts of the snowmobile: A – engine, B – propelling device, C – cab support, D – control, E – back run, etc.

Each characteristic (parameter) has a certain number of different independent features. Thus, the engines: A1 – explosion engine, A2 – gas turbine, A3 – electric motor, A4 – jet engine, etc.;

Propelling devices: B1 – screw propeller, B2 – caterpillars, B3 – snowshoes, B4 – iceblower, B5 – screw feeders, etc.

Cabins: C1 – cab support on the snow, C2 – on the engine, C3 – on the propelling device, etc.

Concerning this problem in the matrix representation (morphological box) the most significant parameters are recorded. For example, for a snowmobile the matrix will be:

(А1, A2, А3, А4)

(B1, B2, B3, B4, B5)

(C1, C2, C3)

Possible combinations: A1, B3, C2 or A1, B2, C3, or A2, B1, C2 or A4, B4, C3, etc.

The matrix is symbolic form of the descriptions of the solutions. It gives an idea of all possible construction diagrams of the snowmobile by fixing one element in each row of the matrix. A set of these elements will represent the possible version of the original problem. Considering the different combinations of these elements, you can get a great combination of all possible alternate solutions, including the most unexpected. Thus, the morphological matrix for jet engines, running on chemical fuel, built by F. Zwicky, contained 576 possible solutions.

Critical stage of the method – evaluation of possible solutions arising from the morphological structure of the matrix. The versions are compared by one or several indicators that are the most important for this technical system.

METHOD OF CONTROL QUESTIONS

This method allows generating the new ideas and solutions, stimulating them with leading questions. It is used in the form of a monologue, addressed to yourself, or the dialogue of inventors.

Page 18: Formulas of Creativity

18

As a matter of fact, it is an improved version of the method of trial and error, as each question is a kind of trial (a series of trials) with one difference: it is easier and faster to cover an initial field of options using the list of the questions.

The authors select from the inventive experience the issues that provide the advantages of the method of control questions over the usual method of trial and error.

One of the most comprehensive and successful lists of the control questions belongs to the British inventor T. Eiloart. For example:

- To find out the opinion of some people, who know nothing about this issue (i.e., to avoid the psychological inertness);

- To arrange a mussy group discussion, listening to every idea without any criticism;

- To test the “national” solution: a clever Scottish, comprehensive German, wasteful American, complicated Chinese, etc.;

- To present the fantastic, biological, economic, chemical and other analogues.

Questions in such a system allow the better seeing of the characteristic of object that is being improved, but how to change it – they don’t suggest it unfortunately.

So, you have learned the several techniques of the search for the technical solutions.

You can get more information in the popular and professional literature [18]. The multiplicity of the methods is explained by the insufficient effectiveness of each of them. In fact they are only the modifications of the method of trial and error, as they allow sorting a lot of versions in a unit of time. But the world is infinitely diversified, and a combination even of a limited number of the elements can be tens or hundreds of thousands.

The current status of the methods of creativity surprisingly reminds the situation, which was at the rise of chemistry as a science. Hardworking alchemists, coming to know the world, found many thousands of chemicals in it. How to make sense of them, how to organize and how to remember them easily? Alchemists were completely confused, although they were offered tens and hundreds of techniques. Actually, each alchemist had his own confidential method. But then it turned out that they could not understand each other anymore... The order was established by Mendeleev periodic table, which showed that there are only several dozens of major primary elements, and the variety of substances is based on a combination of this limited number of elements. This is the world of technology. We have already gone through “the period of alchemy” and our own “Mendeleev periodic table” had to appear. It appeared – a Theory of Inventive Problem Solving – TRIZ.

INTRODUCTION TO TRIZ, OR HOW TO INVENT

I remember an old fantastic story.

The vehicle of the earthlings touched the ground of the distant planet. The flight lasted three years, and the tired crew was curiously glancing at the windows.

Next to the vehicle there were three amazing subjects. They were amazingly reminding the ancient great grandma’s boxes – such iron-sided covers with clinches, shaggy painted sides and huge padlocks. The crew surrounded the boxes and surprisingly started to look at them. The commander contacted the Earth and reported the situation. Confused Mission Control Center said that our great grandmas had never visited this planet and this situation was not

Page 19: Formulas of Creativity

19

provided by any instruction. The commander was recommended to operate at his own discretion.

Let’s interrupt our story here and try to imagine ourselves in the position of this commander. What would you do? Anyone who tried to solve this problem, gave the answers of almost the same type. Many people were thinking that initially it was necessary to open a box and to look – what is there? Have you thought the same? But once the lock has been broken, the box started to disappear, and soon has completely disappeared without a trace. Then only two boxes have remained. What would you do in such situation?

Well, many of you will probably offer to take the boxes in the vehicle and send them to Earth. But as soon as the crew started to lift the box to the vehicle, it suddenly was softened and turned into a liquid, and flew down in the sand. The sample of the sand was quickly taken, but there was found nothing besides quartz. So the last box remained. What would you do?

I am sure that now you took your time to think. And offer to examine more carefully the appearance of the box, not touching it, to take pictures of it, to try to understand its peculiarities, laws of the environment, etc.

This fantastic story ended with the explanation that these so-called boxes represented a form of local life with the highest level of civilization. They came to the vehicle to explore the aliens and to open their content and give all their knowledge if these aliens would appear to be sensible beings. But, unfortunately, the earthlings did not have enough intelligence to avoid the idea that someone might be smarter than them.

Do not we have the same attitude to the world around, believing that we have the right and everything around belongs to us and we will decide how it will look like? The greatest misbelief generated by the human egocentrism. By the way, all the previously mentioned methods have this seal of egocentrism. Let's decipher the word. Egocentrism – a man's attitude to itself as to the center of the universe. In other words, it is the highest form of selfishness. Indeed, while solving the problem, we first think about ourselves and are wrapped up in our thinking. We do not care how do the changed systems “live”, which laws do they obey, how they are interconnected. The real egocentrism!

Ignoring of the laws of the changed systems is the main cause of inefficiency of the known techniques. Psychology as a science of the common factors and forms of the human mental activity cannot help an inventor a lot, as it examines the changes in the inner world of a man, his subjective feelings, not the changes of the objective natural elements.

Philosophers have determined a long time ago that the equipment is a modified element of nature. Indeed, the first hoe, used by the human being, was a folded tree limb, which was convenient for hoeing the soil in search of edible roots. The hoe was modifying, taking a form of a shovel, and a convenient handle. And when the engine appeared, this turned into the excavator – a complex technical system.

And what about our home? Today it is a very complex set of mechanical, hydraulic, electrical and other systems. But initially it was just a cave – a natural element that was modified by the man, who arranged a fireplace, and cut down the sleeping bench and steps, hanged the bed-curtains.

Turning tool was originally an ordinary knife, which was made of stone plates or of the teeth and claws of predators, i.e. again from the natural elements. Such transformations were experienced by clothing, crockery and instruments of labor, in general, everything that surrounds us today and what we call the technical system. Additionally, we can say that the technique is also part of the man himself, or rather a continuation of his organs. Taken stick is – an extension of the

Page 20: Formulas of Creativity

20

hand – has helped to reach an apple that was hanging high on the tree. Stilts, skis, bicycle and car wheels are the continuations of our feet, which make it easier to overcome the distance. Hearing tool that was used by our forefathers, and now the phone, radio – are our “technical ears” that intercept the sounds of hundreds or thousands of miles away. TV is our eyes, brought out to the area where there are some interesting events, etc. Consequently, the technical system is not only the modified element of nature, but also a means of improving human capabilities. As we can see, the objective and subjective concepts are closely interconnected, forming a new, previously unknown world that lives and develops according to its own rules and laws. The theory of inventive problem solving – TRIZ – is studying and using these laws. The principles of TRIZ are simple and clear. They are easily understood, even by the elementary school students. But behind that simplicity is the global experience of thousands of inventors.

Once there was held the competition between the processional designers and students from the inventive circle “Impulse” (Angarsk), in which they learned the principles of TRIZ. Both teams were offered the same task – to design a greenhouse with automatic ventilation. Upon the rise of the temperature the greenhouse had to produce the overheated air, and upon the decrease of the temperature – not to let the cold air. The team of grown-ups has developed their version: the small windows of the greenhouse were connected through a complex system of heavers and bowls with the homing station, which was powered by the signals of the data processing machine and numerous temperature sensors located at various points of the greenhouse. It was a difficult, expensive, but, according to the designers, the only option of the greenhouse with automatic running-off of the heated-up air.

By then the scholars showed their drawings of the greenhouse. There were no mechanisms in their greenhouse, there was no data processing machines and numerous temperature sensors. On the contrary, it has become easier, even the small windows for ventilation disappeared.

- But how is this greenhouse ventilated? – quizzically asked the elder of the designers.

- The greenhouse by itself, when necessary, vents the overheated air and does not let the cold air, – replied the student, one of the project authors.

Indeed, the roof of the greenhouse was made of special transparent porous film, which unclosed its pores when heated and vented the hot air, and closed the pores when cooled, keeping the warm. It's like human skin, which pores enlarge in hot weather and compress when it's cold. Modern chemistry makes it possible to obtain such an artificial temperature-sensitive material.

d (T)

Page 21: Formulas of Creativity

21

Yes, the decision was beautiful and unexpected, so the competition was won by the young inventors.

In this book you will find more than one solution and will go an exciting way with them from the formulation of inventive problem to its solution.

What are the basic steps for the solution of an inventive problem according to TRIZ?

1. System analysis. Identification of the problem from the current situation and its gradual specification through the root cause analysis until it is possible to detect the location of prime cause – the so-called operational zone.

2. Formulation of an ideal final result for the elements in the operational area.

3. Identification of the contradictions that prevent the achievement of the ideal solution. Specification of the contradictions and analysis of their structure.

4. Resolution of the contradictions through the involvement of the laws of technical systems and TRIZ decision tools.

The main TRIZ decision tools:

1. Information fund – a set of heuristic methods of eliminating the technical contradictions, the main principles of resolving the physical contradictions, analogues, physico-chemical and geometrical effects.

2. Su-Field analysis, which considers the interaction and changes of substances and energy (fields) in the system.

3. Standards on the basis of the su-field analysis that are indicating the specific ways of restoring of the functioning of technical system in accordance with the laws of the development of the technical systems.

4. Psychological statements are intended for the depression of the inertness of thinking and the development of creative imagination.

You will learn in detail all these concepts in the sections of this book.

Additionally, TRIZ includes and effectively uses the algorithm of solution of inventive problems – ARIZ. This is a tool that organizes the thinking processes of the inventor in his search for new technical solutions while involving the above-mentioned steps and tools. More details you can find in the literature [4, 5, 6, 10, 14].

And now let's get acquainted with the specific sections of TRIZ.

Page 22: Formulas of Creativity

22

SYSTEM ANALYSIS

While throwing stones into the water, look at the circles formed by them,

otherwise it will be a useless fun.

Kozma Prutkov

INFINITY RANGES

How differently we are thinking while solving one and the same problem! Probably the Georgian philosopher of the 2nd century Ioane Petritsi was right, when noticed: “Every cognizer acts accordingly to his essence”.

For example, we should build a village house. Stove-setter will see the chimney, the carpenter will see the roof, the glazier will see the windows, everyone pays attention on the subjects of his specialization. Probably, it should be like that, if we are dealing with a familiar, repeatedly performed work. But the creative work requires system thinking, which covers the interaction of all systems at all levels.

Let's recall the works of the talented Lithuanian artist Ciurlionis. Most of his paintings are performed, if we are able to find such an expression, in the three-dimensional space, although in one plane. The sea in these paintings is presented to the viewer in the form of the large sea drops located in the foreground, and then they are transformed into the bossom of the sea, and, finally, the endless expanse of the distant ocean surface. Every such picture can be viewed for hours, and it will always seem new as it is so multifaceted and systemic.

And what is the system in technology?

We can give such a definition. “The system is a complex of organized in space and time and interrelated elements that are necessary and sufficient for the performance of the required function, which is defined by the person”.

For example, an airplane – it is a complex set consisting of the fuselage, wings, empennage, engine, controlling tools, etc., which are interconnected and perform a single function – to move in the air. Is it a system? Of course!

And now let us continue the example in a purely relative context. Let us say that as a result of one accident or crack-up the plane went into pieces in the air and its fragments fell on the ground. Does it looks like a system? Of course not. These fragments are not already performing the same function – to fly. But for an expert, who visited the scene of an accident, the lying fragments represent a system, which allows to identify the cause of the accident. Thus, the system is identified by the person, depending on the necessary function.

It is difficult for us to understand or even to remember the spreading of some individual factors or events. But if they make up a logically related plot, i.e., form a system and fulfill the function, we can easily reproduce them at any time. For example, try to remember the following letters: NUCETQHIE. Hard, and the main thing, it is difficult to understand, what for. But the same letters that are arranged systematically and form the word TECHNIQUE, are easy to remember. Our memory is perceiving easier the block concepts, so the effective thinking – it is primarily a system.

Page 23: Formulas of Creativity

23

However, we should clearly understand that there is no isolated systems, neither in the nature, not in the technique. Any one of them is a part of another system, called the supersystem is, and that one, in turn, is itself a part of another, larger supersystem. This range is becoming higher and broader, covering the horizonless areas of space. At the same time, any very small system consists of a number of other, smaller systems, which are called subsystems. And this way depthward, although narrowing, is also infinite and gets lost in the microcosm of the substance. And still all systems are interconnected, and this connection becomes harder while the deepening in the subsystem, and more free while achieving the supersystem. But the interconnection of the systems never disappears completely.

SIGNS OF TALENT

When we talk about the inventor, it is often used a combination “strong thinking”. Then what is the strong thinking? The author of the TRIZ theory G.S. Altshuller answers to this question in the following way: “Casual thinking, when a man sees only what is given in the problem. If the task is, for example, associated with the tree, then a man sees only this tree. Strong thinking – when at least nine mental screens are working at the same time: a man sees the system that is presented in this task, supersystem, subsystem – three different stages. And every stage has past, present and future. So you should see not only the tree, but also the forest and the tree cell. And all that is in the development: past, present and future”.

Solving the problem, the inventor should simultaneously keep an eye on everything that is associated or can be associated with it. Of course, this is a simplified scheme. In fact, as we know, there are many stages up and down from the concerned system, a lot of screens (systems) to the left (distant past) and to the right (distant future). While solving the inventive tasks by trial and error method, one can only accidentally, unconsciously use this type of thinking. But the intuition plays false with us much more often than it is used to say.

Multiscreen way of thinking should become a norm of the inventor, the working scheme, built in the light of the development of technical systems. Let us illustrate this situation with small examples. For example, you are involved in the design of one of the astronaut's life support systems. This system includes a quartz crystal resonator, which looks like a miniature plate,

Page 24: Formulas of Creativity

24

which generates and maintains the necessary signals at the certain level. Your task is to design a special capacity for this resonator, where would be always, in all circumstances, maintained the same temperature. The resonator is very sensitive to the temperature fluctuations, therefore it needs a special protection.

You begin to study the temperature stabilization system. There is a lot of literature and many time tested good devices on this subject. Place the quartz plate in Dewar vessel with double insulating walls, provide the internal heating system, monitoring unit, adjustment unit... Stop! Our quartz resonator somehow became much more complex and heavyweight. From one plate it turned into a heavy suitcase that should be carried by the astronaut. Something is wrong. It happened because we were inspecting the system in isolation from the supersystem, to which it belongs. And what is there? Does the supersystem have the sites, where the stable temperature is maintained? Of course, this is the astronaut himself! The temperature of his body is always 36,5°. That is what we need!

Quartz resonator plate is necessary to put into the inner pocket of a suit of the astronaut – and the problem is solved. In addition, this solution has one more advantage: if the astronaut is sick, i.e., he has the high temperature, the Earth will immediately know this as the frequency of signals will change, and then the measures will be timely taken.

As we can see, the multiscreen way of thinking allows not only to find new solutions, but also to make them more efficient. Once, during the process of construction, where something wrong is always happening, has blown another problem. When the second floor has been already finished and the rugged floor panels for heavy machines have been deposited, it became clear that the autocrane that is maintaining the construction site and handling the pallets with bricks can not reach the third floor. The construction process has stopped, it was necessary to wait for the autocrane with the longer boom from the nearby construction site until it became free. If to take it from there, then people will not be able to work there. The team was not working, abusing of the obtuseness of the workers of the organization management department. The way out of this situation was found by the foreman. At lunchtime he took the large autocrane and used it to adjust his little autocrane on the house under construction. The whole operation took not more than twenty minutes. The work on both sites continued.

What has allowed to find this solution? Systems thinking. The foreman saw not only his autocrane, but also supersystem with its opportunities.

But this story has happened with the honored inventor from Magnitogorsk M.I. Sharapov. He was put in to find the instrument for the prevention of the pipe's wear and tear, which is used for the transportation of the acidiferous wastes. Sometimes it was necessary to change the pipes already in several weeks as the acid was wearing the thickest walls through. The plant was bearing the large loss. The institutions were designing different coatings, but they didn't help at all.

The first thing that M.I. Sharapov has made – he has analyzed the supersystem of dumping of all wastes. He found the other pipe that was located very close and was used for the dumping of alkaline wastes. This pipe, as distinct from the acid, was incrusted so that it had to be cleaned regularly. You have probably guessed, what the inventor has done? Of course, he has connected the pipes cross shaped. Now, in each pipe the alkaline medium was flowing for one week, and acid medium for another week. Alkali has formed the depositions, the acid has removed them. And so on endlessly. Please, note that in both systems almost nothing has changed, but the pipes became wear-free, they still are operating and do not require any repair.

The talented inventor should think systematically, globally, but to act locally and with low costs, considering the future, past and present.

Page 25: Formulas of Creativity

25

THINK ABOUT THE FUTURE

What does it mean to think about the future? Do you remember that Robinson Crusoe, having made the decision to escape from his island, has cut down a huge tree and was hollowing out a boat for several months. Finally, he made it, and only then realized that was not able to drag it to the bank. So was standing his boat away from the bank, accusing its creator of a non-systems thinking. But if Robinson had thought about the future problems, he would have probably chosen another smaller tree or would have rolled over his preparation, until it was still round, closer to the bank and there would have started to make a boat.

Let us remember one of the newspaper articles devoted to the exploration of the Moon. One of the spacecrafts had to obtain a soil sample of the back side of the Moon, in its darkened area. Therefore the ship was equipped with projector for illumination of the Moon area. The operator, who was working on Earth, was observing the Moon's surface and gave the command to obtain some soil samples.

The whole system was worked out and repeatedly tested under terrestrial conditions. There were no doubts of the efficiency of the entire system. The only thing that bothered the designers, was the mechanical stability of the lamp bulb of the projector. When the system was overloaded the lamp bulb failed and broke at the bottom of it. They did not have time to design and test a new bulb as they had only few days before the start of the complex.

They found a corresponding lamp of the tank projectors: they were more firmly carrying the vibration and overload during acceleration, but were significantly heavier than the former. Designers went to the general designer to coordinate their decision. Having found out what had happened, he asked “And why do you need the bulb in the lamp?” – “To protect the glow plug from the interaction with atmospheric oxygen, otherwise it will burn off quickly” – answered the designers. “But is there an air on the Moon?” – the general designer asked again. And then, suddenly, it became clear how much extra work they have done, trying to find the right bulb for the lamp.

That was a result of non-systems thinking, which, unfortunately, is very often present in our actions. Thinking about the future means not to make mistakes in the present.

THINK ABOUT THE PRESENT

The threads of our lives, our actions and our problems are twisted in multielement cloth of the surrounding world. Pulling one of them, we put in motion hundreds of others.

Everything is connected to everything. The ancients said: “Touching the grass do not disturb the star!”

Indeed, if we destroy the vegetation on the Earth, it will change the quality and volume of the Earth's gaseous atmosphere. The atmosphere is likely to be reduced, as the oxygen will disappear that is produced by plants. We can say that the Earth's diameter will reduce, and thus its speed of rotation will increase. Try to remember the rotating figure skater on the ice: the pressing of hands to the body increases the speed of his rotation. That's why the overall dynamics and motion trajectory of the Earth in space will change. And the near-by planets and stars that are interacting with the Earth and the Sun will be forced to dislocate, to regroup, and to balance the whole system. Truly – touching the grass do not disturbing the star!

In this example this situation is intentionally dramatized to visually show the interconnection of all elements, systems, including those in the technique that surround us. Any change of the

Page 26: Formulas of Creativity

26

small part in any car brings the changes of all other parts – in the supersystem and the subsystem.

Everybody knows the wheeled tractors such as “Belarus”. This is a convenient, mobile and multifunction machine in agriculture. It took several years to create of one of its models the lightweight engine made of aluminium alloys. The engine was placed in the serial machine, and then it turned out that the front wheels “hold” the road badly. Lightweight engine was unable to press the wheels tightly to the ground by its weight, and while plowing the tractor in the truest sense of this word “kicked over the traces”. It was necessary to place the range of iron weights in front of the engine on the frame. What have you gained? Nothing. You have only complicated the engine manufacturing technology and have made the entire tractor more expensive. This is the result of non-systems thinking of the designers.

In general, as Bernard Shaw said, “focused specialization in the broad sense of the word leads to a wide idiocy in the narrow sense of the word”.

THINK ABOUT THE PAST

And what does it mean to think about the past of the system? First of all is not to make mistakes in the future. Let us show the example of the above mentioned.

Stationary marine drilling units present a platform that stands on three or four mounting supports, fixed on the bottom of the sea. The platform is high enough raised above the sea level and carries the drilling machines, diesel and compressor units, cabins for maintenance staff and all other necessary equipment. These drilling units are more and more used in the northern seas. But if they are working quite well in summer, in winter there is a lot of problems. The main problem is the freezing of the platform and its mounting supports. Supercooled sea waves form on the mounting supports the large-tonnage ice blocks that should be knocked down every day. It is very difficult and dangerous job. In addition, the ice cover of the sea is constantly “breathing” under the effect of tidal strengths, moving up and down. If it captures the mounting supports of the platform the emergency situation is possible.

There was a competition for the best machine that would knock down the ice from the mounting supports of the platform. There are a lot of constructions with crow-bars, rakes, scrapers and other units. Some of them were even produced. Their only imperfection is the fact that they did not work. The case is that the shearing units in the form of crow-bar were rapidly freezing and uselessly crawling close to the mounting support.

And in one of the American patents it was offered to place around the platform and each of its mounting supports the special machinery in the form of self-propelled rotary tiller. The unit was supplied with powerful electric motors that rotated the giant rotary cutters that were surfacing the ice. Although it was patented, but ugly, energy-consuming and complex solution of the problem. It is known that the ice strength is almost identical to the concrete and to surface it the enormous power will be necessary.

This task was investigated during one of the meetings of the circle of the young inventor “Impulse”. At first all available solutions were considered and rejected, including an American patent. Analyzing the proposed constructions, the guys came to the conclusion that they were “crushing” the effects, not the cause. Thinking over the situation, one of the members of the circle recalled that in early spring when the ice is thin, it is easy to break, and even better is not to prevent its formation on the surface of the mounting support.

Page 27: Formulas of Creativity

27

But for this purpose we need thermal energy. How can we receive it? It turns out that it is in plenty in the supersystem. Diesel engines, standing on the platform and rotating the drill tool, are uselessly releasing the tens of cubic meters of hot emissions into the air. It is necessary just to lower the exhaust pipe into the water near the mounting support – and the problem is solved. The emissions, rising up, not only heat the mounting supports, but, also, while disappearing on the surface of the water, are constantly mixing it, not giving the possibility for the monolithic ice to be formed. Even if the ice is formed, it will be soaked with exhaust gases and will be a completely harmless incoherent mass. And although as a result the engine power, standing on the platform, is less, there is no doubt that the advantage is evident. Exactly this solution by common consent has been formed as an application for an invention.

So, if you want to be an inventor, remember the following:

1. The world is a system and consists of a never-ending ranges of supersystems and subsystems.

2. All systems are interconnected. A change in one of them leads to the change in all others. Hardness (dependency) of the connections increases towards the subsystems and decreases towards the supersystems.

3. Talented human thinking involves at least nine screens – system, supersystem and subsystem in the past, present and future. To see the system in the future means not to make mistakes in the present. To see the system in the past means not to make mistakes in the future.

4. While solving an inventive problem it is necessary to think globally, covering all systems in space and time, and to act locally, with minimal spendings of the space and time.

And now the tasks for an independent decision.

Task 1. You are a librarian. You have hundreds of active readers, who regularly come to take new books. The library building is old, and requires repair. It is necessary to move to a new building, but you have neither cars to transport the books, nor the means to pay for the work of dockers. What to do?

Task 2. You went fishing and had to stop on a country road, as the car got a flat. Common thing. You unscrewed four screws that are fixing the wheel to the axle, and carefully laid them on the side of the road and went to the trunk to take the spare wheel. At this moment the ton up boy, avoiding you, came across the screws, throwing them around. How to find them in the grass?

Task 3. Once a downhill skiing trainer came to an inventor and asked for help. He was preparing a new downhill training lane. According to international rules the surface of this lane should be covered with a coating of ice. The lane is moistened with water. The trainer got a pump, put it on the bank of the river, connected to the electrical line and stretched the flexible tubing to the mountain. But the pump appeared to be underpowered, and the water did not rise above the middle of the lane. Then the more powerful pump was arranged, but the water did not reach the top of the lane again, as it was freezing in the flexible tubing before reaching its end. Which advice would you give to the trainer?

Link to the hints

Page 28: Formulas of Creativity

28

... AND START TO INVENT

Many things are not understandable for us, not because our concepts are weak,

but because these concepts are beyond the set of our concepts.

Kozma Prutkov

Who does not know what harbor he is swimming to,

has no favorable wind.

Seneca

WAY TO THE IDEAL (Law of Increasing the Degree of Ideality)

Those of you, who have visited the Polytechnic museum in Moscow, have certainly drew attention to the “Yablochkov candle”. It was called “Russian luminary” around the world. It was powerfully and evenly illuminating the streets of Paris and London, St. Petersburg and New York. But Yablochkov was not the first who offered to use the arc lamp for illumination. Arc lamps were illuminating also before him, but their light was unstable and capricious. Next to each arc lamp was a servant, who from time to time brought together the ends of the carbon rods that were arranged horizontally toward each other. There was necessary the mechanism of electrode approach as they were burning, otherwise the arc was breaking off.

And this regulator appeared. It was a masterpiece of the design ideas, it was not easier than the famous Kulibin's watch, but, unfortunately, was not so reliable.

Then what has Yablochkov done? He has simply set the electrodes so that... However, I will not deprive you of a pleasure to find the solution by yourselves. Please, listen to such formulation of the task: “The electrodes themselves, without any complication of the system, do not allow the increasing of the gap between their bodies and during the burning of the arc keep it constant throughout the whole length”.

It is likely that having read the task carefully many of you have already solved it. For clarity, I recommend taking pencils-electrodes and trying to arrange them so that the gap between them was the same throughout the whole length. Of course, they need to be arranged in parallel! So Yablochkov did. Now, as the electrodes are burning, the gap between them is not increasing, and always remains constant. No regulator of the electrode approach was necessary, the need for it disappeared. To make the arc still not slipping along the electrode bodies down, Yablochkov has filled the space between them with the ceramic fusing substance.

And now let us come back to the problems of our “car” century. We know how carefully the traffic police pays attention to the condition of the wheels treads. This is understandable. Wheels with worn treads are often the cause of the accident, as they “hold” the road badly and increase the braking distance. Imagine that you have to offer such a wheel, which would alarm that its tread is worn out completely. Lay the book aside for a moment and think about it.

Page 29: Formulas of Creativity

29

Indeed, it is better to lay down under the tread a layer of bright colors. The outstanding bright line along the tire will demonstrate the tread wear. And if to use the phosphorescent paint, the wear will be visible in the dark.

And here's another problem: how to prevent the theft of books from the library. It would be very good if the books were alarming by themselves that they are taken away from the library without permission. Many of you will consider this desire empty. However, one of the British patents proposed to insert into the back of the book the thin magnetic plate or to apply on the back the paint with the magnetic grains. In this case, the book, which is taken away by the plotter, will change the magnetic field strength in the doorway and the sensors will detect this change and will set off an alarm. The book has saved itself! To take it away you will need to go back to the librarian, he will put the book on a table-inductor, where it will be demagnetized and will “allow” taking it away from the room.

Here is another example. In the United States was invented the rat-trap, which does not kill its victim, but only puts on it a collar with a bell. With this “gift” the rat returns to its family and causes a panic. In the result the other rats drive out the “musician” or go away from this house by itselves. The rats drive itselves out.

Examples are enough to notice the common things in them and to make a definite conclusion. It is hard to escape a conclusion that a technical system, being developed, aims to become safer, more simply, more efficient, i.e., becomes more ideal.

It turns out that not only we, the people, want to eradicate our defects and to get closer to our chosen ideal. Our technique also has the need to be improved.

“What could be more powerful!” – we think, looking at the tipping truck that weighs 150 tonnes. “What could be faster!” – we say, looking at TU-154. “What could be more difficult...” – and are anxiously looking at the recently repaired color TV. Yes, the world of technics becomes more complex. And it is not possible immediately to understand in which direction to go in order to improve it even more. But everything in the world follows its own laws. The technical systems also have them. We will talk about them later, but for now let us note only that one of the main laws of any technical system is the law of increasing the degree of ideality. Without following this law it is impossible to improve the technical system.

There may be a reasonable question: if the law of increasing the degree of ideality is so important for the development of a technical system, how could we have managed without it? How we were able to invent thousands of high-performance devices, not knowing about it? And how the ancient people could have built so beautiful constructions, knowing nothing about the building mechanics? How could they have built the fast sailing vessels, which we are admiring today, without any knowledge of hydrodynamics and aerodynamics? It was a difficult and long way of searches and decisions, of trial and error. The hundreds of ships had been constructed until the desired shape was discovered, the thousands of buildings had been built until the desired strength of the material and its laying method was found. Previously, the inventor knew nothing about the law of increasing the degree of ideality, but that did not mean that he did not obey it.

Let us recall the way of the inventor, who is using the trial and error method.

It looks as shown in the figure. Point A represents the initial position of the technical system that needs to be improved. But how to do that? It is not known. One, two or three attempts are made. However, the parameters of the technical system are not improving. We return to the initial position and take a fourth, fifth or sixth attempt in this direction – the result is the same. Finally, the seventh attempt showed that technical system has significantly improved its characteristics, and moved forward. We are at the point A1. For some time there is the successful exploitation of

Page 30: Formulas of Creativity

30

this new machine, but as the needs increase the further improvement is necessary. There is a series of new attempts – 8, 9, 10. From every point there additional shots that are probing the search field.

A1

A2

A3

A

1 2

4

5

789

10 11

1213

1615

17

3

6

21

18

19

20

22

2324

25

26

14

Finally we are at the point 11, and have felt again that the machine has been significantly improved – it became easier, more simply, more reliable. We are at the point A2. There is a new need for improvement, and we are experiencing the same sufferings of another series of trial and error, until we move to the A3.

The scheme shows a small number of trial and error. In practice this number can reach hundreds, thousands and tens of thousands. However, this scheme also shows that some sections are arranged in one line. These sections 7 – A1, 16- A2, 25 – A3 are surprisingly similar to each other and are turned into the same direction. There is the impression that we have from time to time found a good way, which allowed achieving a rapid progress, and that some force pulled us into one side. It really exists, that side, which is called the ideal of this technical system. It is the goal of our machine, which is becoming lighter, more simple, more reliable. Knowing about the existence of such an ideal, isn't it possible to radically change the work of the inventor, that is, not to start from the beginning of the original task, but from its end – with the perfect solution?! Working in the modern way, we should firstly define the ideal of the technical system, and only then deal with its changes. At the same time we allow only those changes that move the technology closer to the chosen ideal. Then the empty fruitless attempts fall down, and schedule of the inventor will appear as shown in the figure below.

We are aggressively and systematically approaching the selected ideal not departing to another side. The ideal for us is like the North Star for the ship, which is running in the vast of the ocean, and even if we do not reach the star, we will choose the correct direction on the Earth. In a sense, the quality of the inventor's work can be assessed by his closeness to the ideal. The greater is the distance to it, the more difficult is to take the machine from the Kuhlman drafting board. And conversely, when the distance to the ideal is decreasing, it becomes easier, safer and cheaper to produce a machine. The ideal is always associated with the maximum use of the real and energy resources, existing in the system, subsystem, and supersystem. The “ideality” of the solution depends on the full involvement of these resources.

Page 31: Formulas of Creativity

31

A1

A2

1 2

4

5

789

10 11

1213

1615

17

3

6

21

18

19

20

22

2324

25

26

14

A3

A

A1

A2

1 2

4

5

789

10 11

1213

1615

17

3

6

21

18

19

20

22

2324

25

26

14

A3

A

In fact, the most perfect solutions are made by nature. It does not have waste as such, everything is used and involved in the never-ending processes.

And what will happen if we reach the ideal? The machine will disappear, but the work will be performed. Remember, please, Yablochkov's regulator. The inventor should reach exactly such “invisible” machine. But as they say: “The plan was smooth on paper, only they forgot about the ravines”.

The way to an ideal is not an asphalt road, on which the inventor can walk, whistling a modern song. Even though this way is a straight line in the plan, it is still chocked up with numerous barriers, the main among them are: the administrative contradictions, technical contradictions, physical contradictions. Art of the inventor is the ability to overcome these contradictions, i.e. the barriers on the way to the ideal.

A3

A

Of course, this is a complex and difficult way, but it is the only possible if we want to qualitatively improve the technical system. We are still expecting a detailed familiarity with all kinds of contradictions and methods of overcoming them.

And now let's try to solve one “sweet” inventive problem. We all love candies. Their production on the line is simple and resembles the production of meat dumplings. From above and below are the sheets of caramel or chocolate, and between them is a solid filling. Candies are produced from this “pie”. But then we have tasted the candy with raspberry syrup. How it got there? Why it has not flown out? The syrup is watery, after all, and cannot be kept between the

Page 32: Formulas of Creativity

32

sheets. You can, of course, pour the syrup into the full chocolate or caramel moulds, but it is very a difficult and time consuming process. How can we solve this problem?

First, let us specify the problem. We have found that all our efforts should be focused on the prevention of the spontaneous flowing-out of syrup. Let's try to imagine the desired ‘ideal’ syrup. This is syrup, which does not flow out, takes any form and may be preserved as long as we need. But syrup is watery and cannot keep the desired shape! We have met a paradox, which is called a contradiction by the inventors – the syrup should be watery and should be hard. What to do? To resolve this contradiction it is enough to have school knowledge. Can the liquid properties be changed? Can it be hard? Of course! It should be just frozen. Here is the answer to the problem. The syrup should be frozen in cubes, and then poured over with watery chocolate. Inside the chocolate box the syrup will melt, and the candy is ready-made. That's it. Is it a difficult task? Not really. However, it remained unsolved for quite a long time.

Now let's analyze a more difficult example, but here the idea of the ideal technical system will be involved.

The flat, very thin and shiny discs arrive to the assembly line over the band conveyor. They have a wet surface – they are arriving immediately after washing, and their shape resemble the ladies' mirrors. One side of the discs is mat – the dark, the other side is shiny. The disks should arrive to the assembly only with the shiny side up, but still there are inverse discs among them, i.e. those that are turned up with the dark side. These are defective discs, and they should be removed from the conveyor belt. It should be noted that the discs are not magnetic or electrically conductive, the use of the photocell will not give a result – it confuses the dark disk with dark belt. The coefficients of friction of the both sides of the discs are exactly the same. Additionally, the discs are afraid even of the touch of dust, so the use of any mechanical droppers gripping devices is out of the question. In addition, the discs arrive to the belt in random order. What to do? Lay aside the book for a while and think about it. If your thinking is too long, do not worry – you are not alone. It is really difficult to solve this problem by trial and error method. Even the professional designers have refused to solve it for a long time, referring to the unusually severe restrictions.

Students from the inventive circle “Impulse” (Angarsk), who found an unexpected decision, were reasoning like that. The inversed discs should be removed from the conveyor belt. Then we compose an ideal of the conveyor belt: “the belt throws off the defective discs”. Here is a contradiction again: the belt should keep the discs and the same belt should throw off them. But

Page 33: Formulas of Creativity

33

not to throw off everything and only those which are laying improperly: there are two tasks – to discover the “wrong” disc and to throw it off.

It is known that any work is performed with the energy consumption. But is there any energy in the conveyor's area? Yes, it is the mechanical energy of motion of the belt itself and the internal energy of the surrounding space, and air.

Then we specify the ideal. “The belt, using the internal energy of the environment and the mechanical energy of motion, discovers the disc, which is laying improperly, and throws it out”. You should not be afraid of such a fantastic perfectionism. Let's try to find a perfect solution, the best in this situation. If you cannot achieve the goal fully, we will slightly depart from the ideal by introducing some new types of energy and substance. By the way, let's specify: which of these objects or substances are present in the conveyor area? The disc, conveyor belt, and drops of water on the disc left after its washing. That's all what we need to solve this problem. Let's recall the physical properties and capabilities of each of these substances during the transmission through them of the heat energy. Do the light and dark subjects react to the heat in the same way? No. The dark discs, while absorbing the heat, warm up, but the shiny discs reflect the heat rays and remain cold. That's why the mirror is always cold, even when lying in a sunny spot. The water boils at 100°C, and increasing its volume, can perform a great mechanical work. Conveyor belt does not change its properties at the same temperature. So we have found one of the solutions.

What happens if we direct to the disc a powerful heat beam, such as a projector lamp? Dark discs will be heated to a temperature above 100°C. The existing liquid under them will boil. These discs like the cars on the steam-air pillow will be tearing around among the cold light discs. The slightest inclination of the conveyor belt – and they will fly off it! Isn't it an original solution?!

And it was found by the 7th grade scholars: Kolya Korenev, Sasha Walter, Dima Domashevsky, Eugene Kelin, Stas Skrynik, Slava Starodubtsev. They also offered some more options. According to one of them the discs should be pre-cooled, so they would be frozen to the conveyor belt. As before, the external heat will affect them. The dark discs will melt faster than the dark discs as they are actively absorbing the heat, and will fall from the inclined conveyor. Perhaps this solution is even more simply and more reliable than the previous one.

You can also cover he belt with the layer, blowing at 80 ... 100°C. Then the defective discs will be in the full sense of the word shoot back from the belt, when being heated. Using the difference in temperature of the discs during the heating, you can find some more solutions. Think about it by yourselves.

Here was given a very brief analysis of the solutions, but it allows to note the main thing- in order to solve any inventive problem it is necessary to study the technical situation, to choose the one specific task in it, to translate it into a mini-task, to define an operational area of the actions, to compose an ideal and to overcome contradictions, involving the resources of the system.

So, let’s remember – any technical system in its development is approaching the ideal, i.e., it becomes more simple, more reliable, more efficient. Upon reaching the ideal, the technical system disappears, but its function continue to be performed.

The moving to the ideal is realized in several ways. Let’s look at them closely.

Page 34: Formulas of Creativity

34

STAGES OF THE IDEAL

The desire to make the machine more ideal – it's not a caprice of the inventor and designer, but a demonstration of the objective law of its development. From the fabulous ancient times we remember the words: magic table-cloth, magic carpet, seven-league boots, etc., which show that our ancestors intuitively felt the promising direction of the development of the surrounding objects. Today it is recognized by us as the basic law of development.

You already know that ideal technical system disappears upon reaching the ideal. It seems a little strange – there is no machine, but the work is still performed. However, there is no mystery. Just the function of the technical system is “plurastically” performed by the super-system, which it includes, or the environment, sometimes it is one and the same.

Today there are a lot of inventions, and they are the pearls of the technical creativity. Look at least at the self-sharpening plough. It is made of three layers: the middle part of the solid material, the edges – of soft material. During the soil treatment the edges of the plough are peeled off faster than its middle part, and in the result it always remains sharpened. And where is the tool for its sharpening? It disappeared. Its function is performed by the external environment, i.e. soil.

There are three main ways of increasing the degree of ideality in technique.

The first way – improving of the multifunctionality of the technical system. For example, after drilling of deep holes in the steel detail there remains chips. There are many different devices for drawing off chips out of the holes. Let’s have the task to improve the ideality of the drill by increasing its multifunctionality. First of all let the drill clean the hole of the chips.

What should be done for that? The first thing that comes to mind is to magnetize the drill and the chips will reach for it. The same drill can control the depth of the center drilling. Depending on the depth of the dipping into metal the variations of the drill are different. Vibratory sensor that is detecting these variations and tuned to a specific frequency, will immediately turn off the machine, when a certain depth of drilling is reached.

The ideal technical system retains its dimensions, weight and consumed energy by increasing the number of functions. This state is clearly illustrated by the modern electronic calculators. Its first generation also performed only arithmetic operations. Today, they can calculate the percents, raise to a power, extract the radicals and perform many other functions, keeping the former dimensions, and even becoming smaller. The same happens with the wrist watch. Some of the models not only show the time, but also serve as an alarm clock, thermometer, pulse meter, etc.

One Japanese company has developed the “watch of the 21st century”. There is mounted a microcomputer, which can present a variety of information, including timetables of the aircraft and trains. This box also includes a vest-pocket radio and a mini-screen for watching TV programs.

All this is possible only under the condition that one and the same element performs dozens of functions.

The second way of approaching the ideal is transformation of the parts of the system in a tool. The process is gradual, long-term, but inevitable. It is demonstrated by any developing technological system. For example, let’s take the car. Its tool, which performs the basic function, is the wheel. So in the vehicle are transformed the elements of the car, which are working for the main function – transmission, the engine, driving. It can be demonstrated by modern heavy vehicles that have motor-in-wheel.

Page 35: Formulas of Creativity

35

As the research of P. Salamatov has shown, the process of idealization is wave-like. Firstly, a technical system, while expanding its functions, becomes more complicated, acquires assistive devices. Then it becomes simpler again, although the number of functions is not less. Then follows a new cycle – a new “expansion” and then “transformation”. Finally there is a complete transformation of the extensive technical system in a single “ideal” substance that is the main tool, and successfully performs all functions.

You probably know that the first TVs were mechanical. Hundreds of electromagnets have built a primitive image on the screen, raising and lowering flags. Then the hundreds of details have “transformed” in an electron beam, embossing the markings on the luminous screen. The number of details decreased, although the amount of transmitted information increased.

New features – channelling, colour grade, and others updated the colour TV with mass of assistive devices that make it work not very reliable. Then should have followed the “transformation” of all its devices that provide colour in a single “ideal” substance. And these TVs have already appeared. They use the liquid crystals that are controlled by an electromagnetic field. Now TV has transformed from a huge, expensive and complicated box into a flat thin sheet, which can be hanging on the wall like a picture. There is only one tool left.

Perhaps the following feature of the TV can be a smell, three-dimensional image, the sensitivity to the mood of the owner, etc. And then it will be updated, expanded again in order to “transform” afterwards in a flat screen, i.e., again tool.

The third way of increasing the degree of ideality – transfer to the supersystem. The technical system develops and then disappears by transferring its functions to an element from the nearest supersystem, thus increasing its multifunctionality. At the new level everything will start from the beginning. There will be a process of increasing the number of performed functions, “transformation” in the tool, and finally the transfer of functions to the next supersystem. In addition more and more subsystem elements of the supersystem are involved in work.

The transfer of functions will be repeated until it reaches the highest supersystem – environment. Exactly the elements of nature will perform all functions of technique... But it can happen only in the distant future, while the process of idealization will be developing for a long time at the level of technical systems.

Let's come back to the TV. It has “transformed” into a tool – flat screen with the performance of multiple functions. What is its next step? Of course, the transfer of its functions to the nearest supersystem, any element of the apartment. Maybe the TV of the future will represent an ordinary window glass. It will be – and it will not be... Our new window can permit the sunlight to the room, convert it into electric energy, and if necessary to become the TV screen. Or maybe the function of TV will not be performed by the window, but by the wallpapers on the wall or lamp shade? In principle, it does not matter. The main thing is that while approaching the ideal the technical system always transfers to the supersystem, at first partially and then completely.

Let’s illustrate this situation by one example. Let’s recall the genealogy of our ordinary pen. How many subjects were needed in the 19th century to write a letter? At least five – a goose quill, a bottle of ink, sandbox for powdering the sheet to dry the ink, the bureau – the high table and a penknife for sharpening the goose quills.

Let’s very briefly retrace the evolution of the writing device, and the way of improvement of its ideality. The goose quill was replaced by the pen with a crow quill. The process of sharpening the pen became unnecessary; the knife also disappeared. From the table it moved to the supersystem, becoming the sharpener for crow quills at the factory. On the table just ink and a pen were left.

Page 36: Formulas of Creativity

36

A new quill pen has become a fountain pen, the so-called unceasing pen. The inkpot disappeared, moving to the floor above, i.e., in the body of the pen. There was only the process of its filling. But this design of the pen, which seemed to be easy enough, was not capable of holding the leading position. It was replaced by the ball-point pen, which was not necessary to be repaired, dipped and filled with the ink. The device for filling the pen disappeared completely transferring to the supersystem – factory. The sandbox also disappeared, it became ideal. At first it was a paperweight, then the special paper. And when the fast-setting paste was developed, the function of taking away of “extra” ink was transferred to the air environment.

Then a number of evolutionary transformations of elements of the pen were performed. In some ball-point pens the refills have disappeared – they have become more ideal as they have transferred their function – keeping of paste – to the closest element of supersystem- the body of the pen. The body also disappears, more and more turning into a tool, which is becoming porous and more multifunctional. New markers represent just one part – sharpened porous refill, penetrated with paste and varnished from outside. The next function is keeping of the paste. The paste should disappear from the pen, and this trend is already up-to-date. The paste should, of course, transfer to the nearest supersystem, i.e., to the paper. The paper, which contains the microcapsules with the paste, is already beginning to be produced in some countries. On this paper you can write by any stick or any hard object. Crushed microcapsules leave a clear trace on the paper.

So, we came back to the old technical system for writing, but at a qualitative new level. The case is that the ancient Romans have used the same sticks, the so-called stylus, for writing on clay tablets. This phenomenon – as it is supposed to be the return to the old – is called the spiral development of the system.

But let's continue the analysis of the pen. What is left for writing? Paper, stick and table. Guided by the law of transfer of the functions to the supersystem, we should make ideal, i.e., “to lose” a stick. Its function should be overtaken by the paper. It not only keeps the ink, but also records our letters – words. Actually, this paper should transform any kind of our sounds into symbols – graphical, magnetic, etc. As you have already guessed, this tape recorder that is designed as a flexible sheet or notebook. We have finally “lost” a pen and a table-desk. There is only one element, which can record, keep and reproduce the words.

In the early 30’s there was an invention which was at first accepted as a curiosity. Its author was B.P. Skvortsov. An invention was called the “talking paper”. The inventor had a task to create a cheap device for voice and musical playback. In fact it was an attempt to make more use of already known method of the photo recording of the sound, but on the plain paper. The invention is significantly ahead of its time and therefore is not so widespread. The magnetic recording, which appeared later, has forced out the idea of photographic recording of sounds on paper. But the process of idealization has not stopped. The modern tape recorders, while developing, will immediately “transform” into one tool – magnetic tape, which will receive the sound signals, store them in a magnetic record and play back at the right time. Imagine that instead of a notebook there is a self-recording paper in your pocket. You do not need any pen, ink and other attributes of recording your thoughts. This idea is not so far from its implementation. Moreover, the possibility of capturing and recording the bio impulses of the brain while thinking is being studied. Perhaps in the future each of us will be able to imaginatively include the recording device and to record our thoughts in a visible image or words.

And who knows, maybe someday there will be created the World Center of Original Thought, where the recording device will be located, which is waiting for the command to receive and record the new thoughts.

Page 37: Formulas of Creativity

37

And then the dream of fantasts became a reality – the creation of the overmind that will be nothing more than a union of creative ideas of billions of people living on Earth.

So, the pen has reached its ideal and disappeared. We have “lost” it, but have not lost the opportunity to make records. However, the replacing supersystem, while improving and approaching the ideal, will slowly lose its elements and, formally becoming more simple, will transfer into a higher supersystem. And this process as it develops, penetrates into outer space. So will be created a Thinking Space, working for a man...

We seem to be a little bit taken with the idea to follow up the law of increasing the degree of ideality by the example of a ball-point pen. Whether this will happen in this way or in a different way, the time will show. As they say, wait and see. The one thing is clear: technique can not be developed without approaching the ideal.

It should be noted another peculiarity of the behavior of technical systems with increasing degree of ideality. At first, the technique is fighting with the forces of nature, and then adapts to them, and finally uses them for its work and development.

Until now, considering the development of the technical system, we have supposed that it is changed by the man. Self-organizing and self-developing technical systems – are still in the future. Then a man will be really busy creating a new class of self-organizing machines and making adjustments to the program of their work, but the machines themselves will increase the degree of their ideality. Today we “manually” turn the technical system in the right direction, and should compose it ideal by ourselves...

We will describe below how to do that.

IDEAL, IDEAL FINAL RESULT AND JUMPING KETTLE

Once James Watt got the idea of the steal engine, while watching at the boiling kettle. Having heard this, the well-known character of the club “12 Chairs” of the “Literary newspaper”, Eugene Sazonov exclaimed:

For forty years I have been looking at the kettle, Poet, novelist, erudite. But it gives rise to none unusual thoughts!

And what about you? Are there any thoughts? No? So it should be. We do not know what we need to invent. As for me, the task with the kettle was suggested by the tragicomical case. As always, in the morning I glanced at watch and realized that I could be late, and hurried out of the house. I was just in time. Half an hour later, having recovered the breath, I suddenly remembered that I left a kettle on a hot plate. The pictures seemed to be more and more frightening... Listening to the comments of my smart colleagues I rushed home. On my way home in the evening I bought a new kettle. Lovingly stroking it and wishing it a long life, I was thinking how to make sure that this kettle will not repeat the story of its predecessor. How should this task be set?

Let's turn the situation with kettle in the task: “Kettle that is left on the hot electric plate does not allow overheating when the water in it is boiled away”. We find in the system “kettle-stove” the conflicting elements: the bottom of the kettle and the surface of the heating element of the electric plate. Now let’s specify the task: once the water has boiled away, the bottom of the kettle should not be heated by the electric plate. Interesting?! Plate is not turned off, then why should not the bottom heat up? So, we came to the technical contradiction.

Page 38: Formulas of Creativity

38

Now let’s imagine a “perfect” kettle. The bottom of this kettle does not allow overheating and shows the ability to protect itself from the surface of the plate. Let's try to get closer to the ideal: the water has boiled away and the temperature of the kettle has risen above 100 °C, at that moment between the bottom of the kettle and the plate appeared a heat insulator. Let’s believe that such situation is possible and let’s look for the best thermal insulator. From the handbook on heat engineering we find out that such thermal insulator is the air. Therefore, in case of overheat between the bottom and the plate there should appear the air gap.

But for that we need to put up the kettle? Let’s specify the task once again – now it is the following: the bottom, while overheating, should put up the kettle by itself. Have you noticed the light at the end of the tunnel? But let’s continue. We should put up the kettle, but we need the energy for that. How can we get it? A short inspection of existing resources – and the energy is found: it is the heat of the electric plate. Now it is necessary only to find an element that can convert the heat energy into mechanical energy. Let’s open the handbook again – there are a lot of such temperature-sensitive elements, for example bimetal: it concaves while overheating. So the task is solved. We should do nothing else than fixing the bimetal sheets to the sides of the bottom of the kettle, which will put the kettle up when heated above 100 °C (water has boiled away). There is a thermal insulator – air gap of 30-40 mm. The kettle is saved. You can place the ring spring at the bottom of the kettle, will be kept by the bimetal fasteners when compressed. Heated above 100 °C, it will release the spring, and a kettle or pot will simple jump from the plate to the floor, saving itself from overheating, and the home from fire.

We should only add that the first people, who have saved the kettle, were 8th grade scholars of the circle “Impulse”, and they received the real inventors' certificates on request “Device for the heat treatment of the food products (variants)”. They are Andrei Usynin, Igor Ivanov, Andrei Motychenko, Gennady Ogorodnikov, Victor Sinyak. It is possible that you can find another solution, but it should obligatory move the kettle to the ideal, because, I suppose that you will agree as well that “perfection is not when there is nothing to add, but when there is nothing to take away”.

“Ask for the moon – and you will get the best!” – believed Napoleon, instructing his generals. We should set a super-task to solve the usual task. Our super-task is the ideal machine, which is the lightest in the world: it weighs nothing, and is the most energy-efficient in the world, i.e. it does not consume energy at all, and finally, it is the smallest –it is almost not visible. But we should learn how to formulate the ideal. Therefore, in TRIZ there are special rules for composing the ideal final result. If the task is free from unnecessary elements or the contradiction is obvious, the ideal final result can be composed immediately, directly for the variable element. If the task is not clear, you should identify the contradiction at first, and then the “guilty” element, and only then to compose the ideal final result. If this element performs the required actions, then take the certain X-element, and provide it will all-powerful properties. Let it not bother you. Solving the task you will step by step determine what lies behind this stranger and how it should look like. General outline of the formation of the ideal final result is the following: X-element, not complicating the system and not causing harmful effects, removes... (to specify the harmful action) during... (to indicate the time of action) and within... (to indicate location and effective area), while keeping the ability to... (to specify the considered element) to make... (to specify the required beneficial action). Of course, the possibility of self-clarification and a change of the formula of the ideal final result are not excluded, if it helps better to show the desired effect.

While solving the inventive problems it comes rarely to achieve the ideal. More often it comes to depart from the set goal and introduce some new elements. But the ideas still work – we get the best.

Page 39: Formulas of Creativity

39

Didn’t you have at school to search for the right answer in the end of the book before solving the task, and then carefully to adjust to the solution? This is, of ‘course, a dishonest way to have fives, but surprising is that this method is the only true and unique in the invention. At first we should find out the answer – the ideal final result, and then to make a decision.

So, let’s briefly state the main ideas that were discussed in the chapter about the ideality.

Technical system in its development is approaching the ideal, i.e., it consumes less energy, time and space for the performance of its function. Upon reaching the ideal, the system disappears, but its function continues to be performed.

The main ways of approaching the ideal: increasing of the number of performed functions, “transformation” into the tool, the transfer to the supersystem.

Upon approaching the ideal, the technical system is struggling with the forces of nature, with the environment, then adapts to them, and finally uses them for its own purposes. Law of increasing the degree of ideality is most effectively applied for the element, which is directly located in the zone of conflict, or gives rise to the undesirable effects.

In this regard the increasing the degree of ideality is usually carried out by using of previously unused substances and energies available in the area of the rise of the task. The farther away from the zone of conflict will be taken the substances and energies, the less you will be able to move to an ideal. But anyway, moving to the ideal, you will experience an ever-increasing resistance in the form of contradictions. You need to know them and to be able to overcome. We will discuss it in the next chapter.

And for now we would like to remind the ancient proverb, which deserves attention of the inventor.

Burnt by the sun of France were shining with sweat the backs of the loaders of the 11th century. The heavily laden with marble carts were loudly rolling along the boardwalk. There was a laying of the foundation for the future and famous Chartres Cathedral.

- What are you doing, dear? – asked passerby the first loader.

- Don’t you see? – he answered. – carrying the stone!

- And you, my dear? – asked passerby another loader.

- I feed my family – was the answer.

- The same question was put also to the third loader.

- But I’m building a palace! – and the man proudly pointed at the foundation.

Perhaps the most important thing in invention is to see the still unbuilt palace.

Using the concept of an ideal, try to solve by you some of the inventive tasks. Do not be confused that they are taken from various areas of human activity. It will be enough to imagine the ideal technical system to solve them. You will be able to find the answers in the end of the book, but take your time before doing that. We remind you that in the end of the book there are some small hints for you.

Task 4. The base of the Pyramid of Cheops has the absolutely precise leveling, however covers the area of 4.5 hectares. How the ancient Egyptians, who had no modern high-precision instruments for levelling, could have done this?

Page 40: Formulas of Creativity

40

Task 5. You are in the dental room. Having placed the filling, the doctor warns you: “If the filling falls out, you should immediately come again to place the new one, otherwise the tooth may be destroyed fully and it will be necessary to remove it”. However, the filling sometimes falls out very quickly, and a person may not know it for weeks.

How to know that the filling has fallen out and to recollect courage for the next visit to the doctor?

Task 6. And now you're a tailor. Somebody has brought a modern bright fabric with different colour strips and asked to sew a summer suit, but so that the seam twists were visible on the suit. You took the order and started thinking. The twist of any colour would stand out on this suit. Hiding of the seam is not allowed by the style. What to do?

Task 7. How to know which of the radioelements is overheated when operated and should be replaced?

Task 8. Automatic moulder produces a round tablet every second. It rolls along the slopping platform and comes into the delivery board of the packaging line. Sometimes the automatic moulder produces a defect: the tablet is not round, but with fractures at the edges, or even broken into two halves. How to separate the defective tablets without using any special mechanisms?

Task 9. When Peter the Great was building a city on the Neva River, the word spread among the working people that this was not a charitable deed. Everybody was referring to the icon of the Holy Virgin, which suddenly started crying when the construction began. Peter threw over his imperial uniform the peasant’s cloth and went to the church together with his Tula armourer, where people were staring at the crying icon. Peter looked at the icon and asked Tychkov: “Is here a great secret?” Tychkov answered “What is crafty, is simple” – and snuffed out a burning candle that was standing by the icon. After this visit, the eyes of the Holy Virgin always remained dry. Which secret has guessed Tychkov?

Task 10. In many large cities the road crossings are equipped with special traffic lights that regulate the duration of the red or green lights, depending on the number of cars at the crossing. How can it be?

Task 11. The sand is delivered to the hopper along the pipe line by gravity. When the hopper is full it is necessary to block the pipe line, but any fastenings or valves are quickly wearing out by sand. Suggest a perfect valve.

Task 12. What are the fundamental changes that will occur in the future with the wrist watch while it is approaching the ideal.

Task 13. You've come to the canning factory. The empty glass cans, washed in hot water, are moving in an endless stream along the conveyor. But some of them have cracks in the body frame and fractures on the necks. Suggest a device for separating the defective cans from the total flow. I only ask you not to dip the can into the water and not to use the photocells and other exotics. Do that simpler, more ideal! Good luck to you on your way to the ideal final result!

Link to the hints

Page 41: Formulas of Creativity

41

IN THE BEGINNING WAS A CONTRADICTION

The contradiction is the criterion of the truth, the absence of contradictions – criterion of the deception ...

Georg Hegel

EMBRYO IN THE NEWBORN (law of the unity of contradictions)

...And the warrior came riding on his horse to the intersection of two roads, and saw that there is a big stone with the inscription: “If you go to the right – will lose the horse, if you go to the left – lay down your life”. So Ivan Tsarevich took thought...

Is there somebody who has not heard this Russian fairy tale? How you listened to it with bated breath a long time ago, truly experiencing and warmly wishing good luck to Ivan Tsarevich? But then you grew up, and you have your own problems, and now you are often standing by yourself at the intersection of the roads, learning the joyless inscription on the stone. And this stone in the inventive practice is called a contradiction. It appears suddenly, like an evil jinn at the full huge length, when somebody wants fundamentally to improve something in the machine or in the production technology. You face it also in everyday life. If you want quickly to cook dinner, increase the flame – please, here it is, this contradiction is acidly looking from the half-burnt porridge.

The world of technology is full of implicit and explicit contradictions. They force the inventor to be the master of his endless search. Immanuel Kant, who was fundamentally judging of everything, said: “To invent something – doesn't mean to open, because something that is being opened, is assumed to be existing before the opening, but not yet known, such as America before Columbus, but something that is invented, such as gunpowder, was known to nobody before the master, which made this invention”.

It is hard to disagree with this idea. Indeed, the inventor has to look for something that does not exist, and even if exists, then for many reasons is not applicable and requires to be changed. By creating a “non-existent” we don't have the rights and capabilities to violate the existing laws of the nature. But we also cannot stop in our development.

In this situation, the famous Hamlet's question “to be or not to be” turns from its rhetorical essence into a severe reality.

If we try to avoid the answer or not to notice this question, it turns into an even more serious problem, which affects more profound public interests and comes into conflict with the environment. The art of the inventor involves the revealing of the key component of the chain of contradictions and cutting-off of this chain by using the known laws of nature.

The use of gunpowder, mentioned by Kant, is also the result of resolution of the contradiction that arose in small arms. Bow as a weapon of war satisfied the attacker until the armor appeared... For the arrow to break through the armor, the bow needed much more power. For this reason it was becoming more and more tight. And not everybody had the strength to pull back the bow-string. There was a contradiction. Initially there was an attempt to overcome this contradiction “straight-forward”. The huge bows were created, which were pulled back by the entire team of archers. But as a result of this action the mobility of the attack units, and the main

Page 42: Formulas of Creativity

42

advantage of the archer – firing speed – was lost. The next attempt to avoid the contradiction was made at the “design” level. It was the same bow, but its bow-string was pulled back already by a hand winch. One archer could have handled this crossbow, but the loading by the platoon took too much time. For this reason this crossbow was not widely developed.

Contradiction in small arms was overcome only after the transition to chemical energy of the substance – gunpowder. The enormous physical strength was no longer necessary, and a bullet, which once was the arrow, has received the sufficient energy to break through any armor. But now this new small arms was left unchanged. It was improved, experiencing new contradictions. For example, the first guns were loaded from the tubes and therefore had to be short, but they also had to be long, as with the short tube the bullet did not manage to gather enough speed from the pressure of the expanding powder gases. The contradiction was resolved when the gun was provided with the bolt and loading cases that allowed putting a bullet in the tube from the side of the club.

The further way of development of small firearms has also been associated with the overcoming of the next contradictions. In fact, while tracing the development of any technical system, the common feature appears to be evident- its qualitative improvement was becoming possible only by overcoming the contradictions. There is no other way.

Apparently, the great Schiller was wrong in saying: “The truth does not suffer, if somebody does not recognize it”. It turns out that it is still suffering. And we will commit a sin against the truth, if while constructing a new technique, will not take into account the emerging contradictions.

Unfortunately, the newcomer inventor, having faced the contradictions in the problem, becomes frightened and shrinks back as it seems to be invincible. But this is equivalent to the situation as if the surgeon in the course of operation would become afraid of the inflamed appendix and sew up the wound not having removed it. The inventor is the surgeon of technique, his duty is to modify or even to remove the “inflamed” host machine. And you cannot become a true inventor, if you do not learn how to identify and overcome contradictions.

Since the mankind passed from the horseshoes production to the cars manufacturing, it has accumulated enough experience of communication with the contradictions in the technology. As a rule, these were the joyless meetings, each of them forced to stop and to choose the way forward. There was a great wish to try to avoid the contradictions. But such actions remind the behaviour of the ostrich, which hides its head in the sand in case of danger, naively believing that now it is in safety.

At the end of the last century London experts have estimated that if the horse-drawn transport continues to be developed under control of the rapidly developing industry of the city, in 20 years London will be under a two-meter layer of manure... This joyless picture was presented by the insolvability of the contradiction – there should be a lot of horses to provide raw materials for factories and plants, and there should not be a lot of horses to save the city. Situation was saved by a car. There was much more horsepower in the city – dozens or even thousands in each engine, while the horses disappeared. The contradiction was resolved by the transition of the transport system to a new level. With the appearance the well-being seemed to be achieved and the city was not in danger anymore. But, as the French say: „if everything is fine, do not worry – it's not for long time”.

Being excited with the cars, in the 20s nobody thought that the unrestrained growth of their number will lead to another problem – air pollution. London was again the first to feel that something is wrong. There was the sadly famous car smog over the city, formed by the exhaust gases of hundreds of thousands engines. People were short of breath, regulators at the crossroads were dressed in oxygen suits. The unmastered technical contradiction in the internal

Page 43: Formulas of Creativity

43

combustion engine made itself known like that: the fuel in the cylinder should burn quickly to give sufficient power, and it also should burn slowly in order to react fully and not to form toxic compounds.

When there was not a lot of cars, this situation did not cause much concern. Power, speed, comfort fascinated the designers. Escaping half-burnt fuel from the exhaust pipe suggested only to establish a more capacious fuel tanks. With an increasing number of cars the situation has changed dramatically. Unresolved technical contradiction came into conflict with man and nature. Various converters of the exhaust gases are considerably complicating the engine and take away a large dose of its power. Different fuel additives are not very helpful as well. They are harmful to the engine itself. By these and similar means we're trying to avoid the contradiction, but not to allow it. The familiar tactics of the ostrich...

Favourite child of the 20th century, having developed, managed to accumulate so much of the unresolved contradictions that have been fighting with a man for a long time, punishing him for his poverty and shyness of the inventive idea. Now, over a quarter of a million people are dying in the car accidents every year. It's more than in all natural disasters of the world – earthquakes, hurricanes, storms, etc. The car, disturbing the ecological balance and applying pressure on the nature, causes far more damage than all other invaders and disease causing microbes. If the nature was able to shoot, it would firstly „shot down” the car.

Contradiction, which is carried by an internal combustion engine, turns into a worldwide disaster. This situation cannot stay longer. It should be changed. And it will be changed when we learn how to burn the fuel in the cylinder without the formation of toxic emissions. But for that we need to dramatically increase its burning time, what will lead to a loss of power... One more contradiction.

Try to resolve it, using the recommendations of TRIZ, and divide the conflicting demands in time and space (see Appendix 1). It means that the fuel should be burned, completely and qualitatively, in one place, and to use that gained in another. It is necessary to release the vehicle from the internal combustion engine, arranging the power plant – electrical, inertial, gas, etc. We think that the inertial accumulator will be the first. It is simpler, easier and more reliable than electric. Flywheels can store up the energy that is sufficient for the motion of the car for hundreds of kilometers. Constructions of flywheels are being improved. Once hyped up in an evacuated jacket they may maintain their rotation for many months.

The picture will be real when instead of the engine in the car will be installed the flywheel, hyped up at full speed. Its energy will be enough for a few months of car operation. Then will follow a new backspin, and the car will drive without an engine. For hyping of the flywheel the special thermal power plants will be provided, which essentially represent the same car engines, but brought away outside the city and combined into one. Thus, there is a real possibility to store up the energy in the flywheels from the low-level renewable sources – wind, geothermal waters, solar heat, etc.

So, the car will be ecologically clean. What’s next? Will all the problems disappear? Unfortunately, after the further development of the car the new contradictions will appear, which would also require a resolution. This process is endless, it is dictated by the dialectics of development. Any technical system carries the beginning of the future problems, which increase with our needs.

It is necessary clearly to understand – contradictions in the technology appear only when we increase our requirements for a particular technical system or any of its part. Moreover, the requirements for an object can be very different – thus the multiplicity of contradictions appears. The close interaction between desired and undesired has been noted already in ancient times.

Page 44: Formulas of Creativity

44

The Chinese monad interpreted this dual connection as follows: „There is no good, which would not contain the evil and no evil, which would not contain good”. Graphically, it was expressed in the following way.

Of course, you can agree or disagree with these generalizations, but the inventor should see in this newborn car the embryos of the future defects. This allows taking timely actions to resolve contradictions even before they loudly declare themselves.

Let's admit that it was possible to design a car, which is capable of gaining the speed of 700...800 km/h. What will the contradictions? One of them – the brakes: the reliable, “dead” brakes are necessary to brake hard in case of danger, and you cannot have them, as the hard braking can cause overloads that will not be stood out by the passengers. The way out: to provide the car with a special radar installation, which would probe the road a few miles ahead, as the braking distance is more than 500 meters. But all this will incredibly complicate the car, and besides, any road has turnings, batters, where the radar installation will be useless. The revealed contradiction makes abandon the planned car. But people want to drive faster...

What to do then? The contradiction, found in the planned car, can only be resolved at the level of supersystem, to which it belongs, i.e. it is necessary to change the road itself. Its blade should be raised above the ground to avoid the appearance of any random object on it, which would cause the need for emergency braking. Most likely, this road will resemble the monorail, which serves not only as a support for the wheels, but also allows transportation of energy for motion. And what's left of the car? Cabin for the passengers and the engine of electrical or electromagnetic type. The goal is achieved – you can safely drive with the speed of 700...800 km/h.

Overcoming the contradictions, the urban transport will transit from the monosystem, which is represented by some cars, into the polysystem. These will be the already mentioned monorails and flexible high-speed sidewalks, and pneumatic tube transport, hidden under the ground, and other equally highly organized, environmentally clean and safe technical systems. Of course, the possibility of using an ordinary car is not excluded. But it will be a promenade, special or sport car, relatively low-speed and so rare, as a horse on the streets today.

It follows that the contradictions not only define the present but also the future of the developed by the technical system. Identification and analysis of the contradictions form the basis of the prediction of the technology of the future.

It is interesting to note the remarkable invention of nature, which it demonstrates in case of contradictions. Here is one example. Lost in the vast of ocean island of Mauritius is covered with the jungles of calvaria. But the seeds of these plants are pecked by the dodo birds. Nature found the following way out. It provided the seeds of calvaria with unusually thick shell. The embryo of a plant can only hatch out if the shell of the seed has nuzzled against the digestive stones that dodo, like many other birds, swallows and constantly keeps in the stomach. By

Page 45: Formulas of Creativity

45

resolving the contradiction – to be eaten and not be eaten, calvaria obtained the possibility to multiply and prosper.

Now, after we have “painted” a general portrait of the contradictions in the technology, let's look closely at its shape.

Among the chaos the three types of contradictions are clearly distinguished – administrative, technical, and physical.

Let's get acquainted with each of them separately.

But before that try to practice in finding contradictions of common types and in overcoming them on the basis of the tasks.

Task 14. One of the Seven Wonders of the World – Lighthouse of Alexandria in Egypt, Mediterranean coast. Time has destroyed the lighthouse, but archaeologists say that it was more than 300 m tall.

The lighthouse was standing for several centuries with the inscription at the top: “Sostratos of Dexiphanes the Cnidian to Saviour Gods for the seafarers”. That was the name of the builder, and people always remember his name. But the history remembers also something different. When the construction of the lighthouse was ended, the emperor called Sostratos and ordered: “You will record my name on the lighthouse!” The builder knew – if he fails to fulfil this order, he will be executed, and if he fulfils – the descendants will never know the name of this author of the lighthouse. The builder was kept alive, but the whole world knew his name. How could this happen?

Task 15. There was a all-mighty devil. One day he caught three travellers and agreed to let them go if they set an impossible task.

The first traveller asked to make a tree of gold, the second – to turn the river back. Both tasks were easily performed by the devil.

A third traveller was kept alive. Which task did he set?

Task 16. Which contradiction exists in an apple fallen from the tree, and how it is resolved by nature?

Task 17. Which contradiction is not resolved in the pike of the building poling?

Task 18. What are the home products that are simultaneously hot and cold, unified and crushed, edible and inedible, solid and liquid.

Task 19. What is the creature that simultaneously flies with the speed of sound, and with a speed of pedestrian.

Task 20. How to eat something that is known by everybody seen by nobody?

Link to the hints

If some tasks are difficult for you, do not give up. Come back to them after reading the chapter “Duel with the contradictions”, and you will succeed for sure.

Page 46: Formulas of Creativity

46

I KNOW WHAT, BUT I DON’T KNOW HOW (administrative contradiction)

The philosophy is the administrative contradiction is simple and sounds disarmingly straightforward, for example: "It cannot be done, but still needs to be done!” or “It can no longer continue, we should do something!"

Convincing, isn’t it? Administrative contradiction, are usually created by the man himself, or rather by the organizational circumstances that he created. Let’s describe the following historical event as an example.

About twelve hundred years ago, in 800, the coronation of Charlemagne took place. In accordance with the ritual the crown on the head of Charlemagne should have been set by the Pope. Charlemagne faced a difficult task. Coronation was necessary for the consolidation of power, so the political considerations dictated the need for its implementation “with due ceremony”. On the other hand, because of these political considerations, it was totally unacceptable that the Pope crowned Charlemagne, as it turned out that the Pope was above the emperor: if the Pope gave the crown, he might have taken it away one day. There was a complicated situation: Charlemagne should have been crowned by the Pope in order to keep the ritual, and should not have been crowned by him in order not to be dependent on the clergy. Charlemagne found an original way. At the time of the coronation he snatched the crown from the Pope’s hands and placed it on his head. Long live the King! Nothing and nobody can be above the King!

A thousand years later, in December 1804, in Notre Dame, Napoleon's coronation took place. And there was again a similar situation. Napoleon used the method of Charlemagne, and also snatched his crown from the hands of the Pope...

And what would you do? Contradiction is the same, so the method must be the same.

Administrative contradiction is also in technology. However, there it is a bit different, but still connected with the man and sets him thinking – it is necessary to do something, but I don’t know how.

Administrative contradiction includes a tangle of problems, and many difficulties are difficult to be resolved because of the attempts immediately to understand the situation, without identifying a specific problem. The unpopularity of this original list of inventive problem is defined by the fact that there are usually presented the problems at the level of administrative contradiction. The duties of the inventor include the overcoming of this contradiction by transformation of unclear, muzzy situations, consisting of a tangle of problems, in one particular technical problem.

Let's look at one of these lists and try to overcome the existing administrative contradiction there by an example of such a problem. “Improve the existing high-voltage power line, mast supports that are located in the permafrost zone. Some foundations of the mast supports warm the adjoining soil, which deliquates, loses its bearing resistance, and supports fall. Suggest a way that excludes this fact”. After reading this statement, you can hardly imagine what should you do, in fact.

To develop a new mast construction that will not transfer heat to the foundation? But the mast is already constructed and is located, where it should be. To change the mast is no longer possible. Suggest a new foundation? This cannot be done as well. This would require significant capital works and line disconnection. To change the soil around the foundation? It is unlikely that the line disconnection will not be necessary. So it turns out that you can only strengthen the existing soil by its artificial slight-freezing. But there are hundreds of masts. Is it possible that it

Page 47: Formulas of Creativity

47

will be necessary to treat the soil around each mast with the low-temperature liquid? It is impossible and not needed. Emergency defrost of the coat occurs only under certain mast supports. But how to know, under which is it? Here is a specific problem. It is formulated as follows: “Suggest a device or method for detection of an emergency defrost of the permafrost soil under the supports of high-voltage power lines”.

Compare this with the original wording. As you see, the difference is great. Now you already know which problem should be dealt with.

In case of re-design of the line a different direction could have been discovered, such as creation of self-cooling supports. But this approach is not applicable to the already constructed line. It is necessary to proceed from the current situation and to choose from the existing problems the one whose solution is implementable.

Thus, the main features of the administrative contradictions in technology – uncertainty of the situation, the conflict between man and technology, the appearance of new needs in the absence of its implementers, or the inability of the technology to satisfy the needs of the old equipment, but to a greater extent.

Overcoming of an administrative contradiction, i.e. the selection of a specific problem in the specific conditions is associated with the conduction of the root cause analysis of the situation, search for the primary cause. Later you find out more information about this work, but now let me recall that, having overcome the administrative contradiction and discovering the problem, you will inevitably encounter a new, so called, technical contradiction.

I KNOW HOW, BUT THIS MAKES EVERYTHING EVEN WORSE (technical contradiction)

Technical contradiction arises between the parameters of the system, its components or groups of details. It dictates: “If you improve one thing – it will certainly makes worse the other!” A lot of pegs with the tablets “finish” was left here by the inventors.

The following example will help you to understand the essence of the technical contradiction. For example, you have decided to increase the speed of the airplane and installed powerful new engines. A terrible roar of the engines shakes the airdrome plates, but the wings cannot tear off the much more heavier airplane from the ground. Then you have decided to increase the wings. Now the plane takes off, but the increased wing drag consumes the power of your new engines. The goal is not achieved – the speed has not increased.

This happened because the main contradiction was not resolved: the wings should be large enough to provide sufficient lifting force on takeoff, when the speed is still small, and they should also be small in order not to cause the large head resistance during the high-speed flight, when the lifting rises sharply.

Until recently it seemed that this contradiction is insoluble. The designers agreed to a compromise. They chose the minimal area of the wings such, but sufficient for takeoff. But it was an alternative route, of two evils was chosen the least. For some time the situation was tolerable. But then the new, higher speeds were necessary. It was not possible to follow the beaten path anymore. So they decided to increase the take-off and landing speed of airplane, raising it to several hundred kilometers per hour. It was more and more difficult and dangerous to boost such airplanes into the air and to land them on airfields. Takeoff and landing became one of the most complicated and important elements of the flight. The length of the take-off runways was measured in kilometers. The airports were located farther and farther outside the

Page 48: Formulas of Creativity

48

city. There was a situation when the way to the airport took more time than the flight itself. There was a patchwork as the result – one thing was improved, another was unacceptably worsened. This contradiction with the wings required a resolution. And then there appeared the planes with a variable-sweep wings. Now on takeoff the wings were “expanding”, increasing their area by 50...60%, and with increase in altitude and speed, they again decreased their area. This allowed significantly to increase the cruise speed with the same engines.

Lets’ consider a more earthly situation and come back to the problem of signalling of the soil defrost under the supports of the powerline. You remember that we need to find the support under which there was an unacceptable emergency soil defrost to have time to take appropriate measures for its artificial slight freezing. Methods of slight soil freezing are known and widely used, for example, its treatment with liquid oxygen or nitrogen. Considering that the supports of the powerline are often located along the mountainous or swampy areas, it is desirable to know the status of soil from the air, for example from an airplane or helicopter.

The first thing that probably comes to each of you – to put in the soil near to the foundation several thermometers. Taking the readings and transforming them into the signaling lights on the mast, we could judge of the state of the soil from the air. It seemed to be the only right way. But ordinary thermometers are not applicable: buried in the soil, they will not provide information. Consequently, it is necessary to apply electric contact thermometers or special thermal couples. But then will be needed the individual sources of energy in the form of batteries, additional devices for receiving and processing of received signals, special electric lines and controlling equipment. Moreover, considering the frost rending of the soil, it will be necessary to take measures not to damage the included and wires. There is a need for special protective devices and facilities.

So, wishing to improve the informativeness of the system, you are faced with unacceptable complexity. Really: „I know how to do, but this makes everything even worse”. Then you decide that it is better to leave everything as usual, and with the help of a crowbar or shovel to determine the condition of the soil around each support. However, will take a long time and a lot of money, but this is reliable. But, judging like that, we thus recognized the defeat from the identified technical contradictions. What to do? The special set of techniques of resolution of technical contradictions available in TRIZ will help to find a way out of this situation. These techniques and rules of their use will be described in a separate chapter.

It is necessary to remember: the technical contradiction is a conflict within the technical system, between its subsystems. While improving one thing another thing is being unacceptably worsened.

Let’s consider another real manufacturing situation, which will be an example of the development of technical contradictions.

Page 49: Formulas of Creativity

49

The excited crane operator entered the office of the department head and said: „I refuse to work in your production department. I absolutely can’t breathe there! My cabin is at the top, and all the smoke comes there from the hardening bathes.” The department head calmed the woman as he could, and then was thinking and analyzing the situation. This was not the first time that the workers complained of excessive air pollution.

The order for thermal hardening in oil of the set of large parts was received unexpectedly, and a special area for this operation was not ready. It was necessary to realize thermal hardening right in the production department. Bridge crane took an overheated in the heating stove part and dipped it in a bath of oil. While dipping of the part the adjoining oil flashed, and the thick harsh smoke filled the production department. When piece was dipped in the oil completely, the burning stopped, only a small stream of smoke of evaporation rose above its surface.

Method of hardening was not allowed to be changed – it was prescribed by the technology, but it was also impossible to perform such work in the production department. Already familiar administrative contradiction. Existing ventilation was unable to cope with high gas content. We tried to increase its power, but new vent pipe blocked the whole department, creating such draft, that many workers were getting cold. So it was necessary to abandon that idea. We decided to install the cover over the bath, which would spring after the descending part. But, according to the rules of safety, the crane operator should always see the goods with which he works, and the cover disturbed this process. Therefore, this innovation had to be abandoned.

There was announced the commercial competition, how to reduce the gas content in the production department. The first place took a decision, which proposed sharply to increase the rate of descent of the overheated part in oil. Urgently was given the task to designers to redesign the crane headgear construction in order to increase the time of the dipping of part from 2 seconds to 10 seconds. The crane was redesigned in two days. The test was being conducted. The part slumped into the bath, raising great waves that slopped over the sides. It was necessary to increase the height of the bath, but it created a lot of discomfort for the maintenance staff, who was hooking and unhooking the dipped part. Additionally, it was discovered that the brake system of the crane could not sustain the new loads, and the while the next dipping of the part the bottom of the bath was broken through. The production department was flooded with oil. Then the mechanics installed the new powerful brakes which immediately stopped the goods. The dynamic loads during the hard braking led to the deflection of the load beam of the crane. The production department stopped.

The chief engineer called an emergency meeting of engineers and technicians to find a way out of this emergency situation. Technical contradiction was more and more escalating. The Council decided immediately to strengthen the crane bridge girder with the stiffening girder. And did that. Two days later it was discovered that the crane girder could sustain the dynamic loads and did not deflect, but in the rail supports, on which the crane stood, appeared the cracks... It could have caused the crash the accident in the production department. What to do? The situation was hopeless.

As you see, the unresolved technical contradiction, wherever it may be, in any case leads to a sharp deterioration in the adjacent nodes, and more and more drawing away in the supersystem, causes the dangerous violations. You have probably guessed what was the fundamental error made by the winner and the designers of the production department. They were struggling with the result, rather than with the cause. The technical contradiction is a signal that the chosen task is wrong and that it is being solved in the wrong place.

Try to get back to the original background of invention and see its primary cause. Thus, there is a smoke pollution of the production department. Until the overheated part doesn’t touch the oil its burning does not occur, i.e. the problem does not exist. When the part is completely dipped

Page 50: Formulas of Creativity

50

in the oil, there is no problem as well. Troubles occur at the moment of contact of the parts with the surface of the oil. That is the problem. Now let’s define the basic elements in a given place and at the given moment: it is a hot part, oil and air. From these elements we choose one that causes the adverse event. The source of the smoke pollution is the oil. But we cannot do anything with it, because it provides the main technological process, which cannot be changed. And what causes the firing of the oil? Air, or, rather the oxygen contained in it.

Let’s accomplish the analysis of the problem of the part hardening for now. Try to find a solution by yourselves. We will come back to this problem in the next chapter. Now let’s select the main thing: the farther is the solution of the problem from the primary cause of the rise of adverse event, the more acute and insoluble will be a technical contradiction. Of course, it can be conciliated by making the whole system more complicated, but as a rule, it soon makes itself known again in another place. The technical contradiction – it's just the middle on the way to solution of the problem.

To avoid a “chain reaction” of adverse events that appear due to the technical contradictions, it is necessary to specify the time and place of the primary causes and to discover the physical contradiction in it. We will get acquainted with it in the next chapter, but for now try to solve the problem. Perhaps you will not be able to overcome these problems without a pause. Then read the chapter about systems thinking and ideal technical system again. If the good success does not attend you, keep on reading the book and come back to the problems after the chapter “Duel with the contradictions”.

Task 21. The one meteorological stations required dipping into the water-hole the devices for parametering of the water and sampling four times a day in winter. The work was complicated by the fact that the hole was freezing after 2-3 hours it was necessary to break through again by the crowbar. There was no complicated mechanisms at the meteorological station. How to release the employee of the meteorological station from manual work on breaking through of the water-hole?

Task 22. The heavy press of 40 tones was rolled in by tube rollers to the production department, and put above the pit of the foundation, where it should have been moved down, removing the rollers. The pit depth was 0.5 m. There was no suitable lifting equipment. Installation of the press stopped. The plan of the launching of the production department, scheduled for the first quarter of the year, collapsed. What to do?

Task 23. You are holding the welding lamp that emits the infrared (thermal) beam. With it you need to connect with a complex joint the two transparent thermoplastic films. Move the beam carefully and accurately, it is difficult to achieve. As a result you lose a lot of time, and the productivity is low. In order not to overfire the film reduce the beam power as much as possible, but then the welding speed is even more reduced. How to increase the preciseness of the welding considering your “inefficiency”? The technical contradiction: the radiation power sharply decrease the preciseness of manufacturing.

Task 24. Workers obscure the joints between the concrete blocks of the foundation with lime, sand and cement with water. In this narrow deep channel of 2 meters fell down a nestling. How to get it?

Task 25. How to replace the damaged area of the pipeline system, not draining the water?

Task 26. How to determine the wear limit of drilling tool, located deep in the borehole, into which the flush water is conveyed?

Link to the hints